[go: up one dir, main page]

Download as pdf or txt
Download as pdf or txt
You are on page 1of 67

KENDRIYA VIDYALAYA GACHIBOWLI, GPRA CAMPUS, HYD-32

SAMPLE PAPER TEST 01 FOR BOARD EXAM 2023

SUBJECT: SCIENCE MAX. MARKS : 80


CLASS : X DURATION : 3 HRS
General Instruction:
1. This question paper consists of 39 questions in 5 sections.
2. All questions are compulsory. However, an internal choice is provided in some questions. A
student is expected to attempt only one of these questions.
3. Section A consists of 20 objective type questions carrying 1 mark each.
4. Section B consists of 6 Very Short questions carrying 02 marks each. Answers to these questions
should in the range of 30 to 50 words.
5. Section C consists of 7 Short Answer type questions carrying 03 marks each. Answers to these
questions should in the range of 50 to 80 words
6. Section D consists of 3 Long Answer type questions carrying 05 marks each. Answer to these
questions should be in the range of 80 to 120 words.
7. Section E consists of 3 source-based/case-based units of assessment of 04 marks each with sub-
parts.

SECTION – A
Questions 1 to 20 carry 1 mark each.
1. The diagram shows the reaction between metal and dil. acid. What is the reason for different
behaviour of Mg in test tube B?

(a) Mg is lighter element than dil. HCI.


(b) Mg reacts with dil. HC1 to produce H2 gas which helps in floating.
(c) Mg reacts with dil. HC1 to produce N2 gas which helps in floating.
(d) Mg reacts with dil. HCI to produce CO2 gas which helps in floating.

2. What is the difference in the molecular mass of any two adjacent homologues?
(a) 14 amu (b) 15 amu (c) 16 amu (d) 17 amu

3. What is the focal length of a plane mirror?


(a) Infinity (b) Zero (c) 1 (d) + 1

4. The diagram below shows a leaf that was covered by a piece of black paper for a period of 3
days. After 3 days, the paper was removed. On testing, it was found that the area under the black
paper tested negative for starch and the rest tested positive for starch. What was the experiment
trying to test?
(a) If plants make their own food
(b) If light is required for plants to make food
(c) If plants can respire in the absence of light
(d) If plants can survive even in the absence of light

Prepared by: M. S. KumarSwamy, TGT(Maths) Page - 1 -


5. A current-carrying conductor is held as shown:

In which direction should current be passed in the conductor in order to produce a clockwise
magnetic field around the conductor?
(a) North to South (b) South to North (c) East to West (d) West to East

6. For a current in a long straight solenoid N-pole and S-pole are created at the two ends. Among
the following statements, the incorrect statement is:
(a) The field lines inside the solenoid are in the form of straight lines which indicates that the
magnetic field is the same at all points inside the solenoid.
(b) The strong magnetic field produced inside the solenoid can be used to magnetise a piece of
magnetic material like soft iron, when placed inside the coil.
(c) The pattern of the magnetic field associated with the solenoid is different from the pattern of
the magnetic field around a bar magnet.
(d) The N-pole and S-pole exchange position when the direction of current through the solenoid
is reversed.

7. At the time of short circuit, the electric current in the circuit:


(a) vary continuously (b) does not change (c) reduces substantially (d) increases heavily

8. The graph below shows the variation of force acting on a conductor with current:

Prepared by: M. S. KumarSwamy, TGT(Maths) Page - 2 -


After analyzing the graph, a student noted the following. Select the correct statement:
(a) The force acting on a conductor increases exponentially with increase in current.
(b) The force acting on a conductor decreases exponentially with increase in current.
(c) The force acting on a conductor increases linearly with increase in current.
(d) The force acting on a conductor decreases linearly with increase in current.

9. In a study it was found that fused ear lobes were found in more numbers within a population
rather than free ear lobes. What can you infer from the above observation with respect to
dominant/ recessive trait?
(a) Fused ear lobes – dominant (b) Free ear lobes – dominant
(c) Fused ear lobes – recessive (d) Both are dominant

10. What is the minimum resistance which can be made using the following resistors?

(a) 1 Ω (b) 2 Ω (c) 4 Ω (d) 3 Ω

11. In the given diagram, when the magnet is pushed into the solenoid, the pointer of the
galvanometer deflects slightly to the left. Which of the following changes would cause the
pointer to deflect through a larger angle?

(a) Move the magnet faster. (b) Move the magnet away from the solenoid.
(c) Unwind some of the turns of the solenoid. (d) Keep the magnet stationary.

12. Manish’s mother was baking cake in the kitchen. When Manish came back from school, he
detected smell of hot cake from the drawing room. Why?
(a) Due to the presence of olfactory receptors in forebrain
(b) Due to the presence of taste buds
(c) Due to the presence of olfactory receptors in midbrain
(d) Due to the presence of olfactory receptors in hindbrain

13. Which of the following is not the role of decomposers in the ecosystem?
(a) They clean the environment.
(b) They decompose non-biodegradable substances.
(c) They participate in food chain.
(d) They replenish the nutrients in the soil.

14. In the given reaction : ZnO + C → Zn + CO.


I. ZnO is being oxidised.
II. CO is being reduced.
III. C is being oxidised.
IV. ZnO is being reduced.
Choose the correct statement.
(a) I and II only (b) III and IV only (c) I, II, and III only (d) All of these

Prepared by: M. S. KumarSwamy, TGT(Maths) Page - 3 -


15. Common salt besides being used in kitchen can also be used as the raw material for making :
(i) washing soda (ii) bleaching powder (iii) baking soda (iv) slaked lime
(a) (i) and (ii) (b) (i), (ii) and (iv) (c) (i) and (iii) (d) (i), (iii) and (iv)

16. A straight wire is placed between two poles of a magnet as shown in figure. If an alternating
current passing through a wire then wire will

(a) Move into the page only (b) Move out of the page only
(c) Move out and into the page (d) Remain stationary

Q. no 17 to 20 are Assertion - Reasoning based questions. These consist of two statements –


Assertion (a) and Reason (R). Answer these questions selecting the appropriate option given
below:
(a) Both A and R are true and R is the correct explanation of A
(b) Both A and R are true and R is not the correct explanation of A
(c) A is true but R is false
(d) A is false but R is true

17. Assertion (A): In Fleming's left hand rule, the direction of magnetic field, force and current are
mutually perpendicular.
Reason (R): Fleming's left hand rule is applied to measure the induced current.

18. Assertion (A): Unisexual flowers have separate male and female flowers.
Reason (R): Cucumber, pumpkin and watermelon are the examples of unisexual flowers.

19. Assertion (a): Non–biodegradable substances are those substances which cannot be broken
down into simpler harmless substances in nature.
Reason (R): Non–biodegradable substances can cause air pollution and make the air poisonous
when burnt.

20. Assertion (a): The effect of root pressure in transport of water is more important during
daytime.
Reason (R): Transpiration pull is the major driving force in movement of water during the day.

SECTION – B
Questions 21 to 25 carry 2 marks each.

21. Identify the displacement and the double displacement reaction from the following reactions.
(a) HC1(aq) + NaOH(aq) → NaCI(aq) + H2O(l)
(b) Fe(s) + CuSO4(aq) → FeSO4(aq) + Cu(s)
OR
A teacher provided acetic acid, water, lemon juice, aqueous solution of sodium hydrogen
carbonate and sodium hydroxide to students in the school laboratory to determine the pH values
of these substances using pH papers. One of the students reported the pH values of the given
substances as 3, 12, 4, 8 and 14 respectively. Which one of these values is not correct? Write its
correct value stating the reason.

22. (a) Which plant hormone is present in greater concentration in the areas of rapid cell division?
(b) Give one example of a plant growth promoter and a plant growth inhibitor.

Prepared by: M. S. KumarSwamy, TGT(Maths) Page - 4 -


23. (a) What is the unit of current? Express it in terms of charge and time.
(b) The following table gives the value of resistivity of some materials:
Material Resistivity (Ohm-m)
A 44 × 10–6
B 1010 – 1012
C 1.62 × 10–6
D 1015 – 1017
Which material would you suggest to be used in electric heating devices? Give reason for your
choice.
OR
(a) On what factors does the resistance of a conductor depend?
(b) Calculate the resistance of an aluminium cable of length 10 km and diameter 2.0 mm if the
resistivity of aluminium is 2.7 × 10–8 m.

24. What is a rainbow? Draw a well labelled diagram to show the formation of a rainbow.

25. What are the differences between the transport of materials in xylem and phloem?

26. Give reasons for the following observations:


(a) Covalent compounds are poor conductors of electricity.
(b) Highly reactive metals cannot be obtained from their oxides by heating them with carbon.

SECTION – C
Questions 27 to 33 carry 3 marks each.
27. Give reasons for the following:
(a) Ionic compounds have high melting and boiling point
(b) Ionic compounds conduct electricity in molten state
(c) Ionic compounds are solid at room temperature and are somewhat hard.

28. (a) What is a solenoid?


(b) Draw the pattern of magnetic field lines of (i) a current carrying solenoid and (ii) a bar
magnet.
OR
Suppose your parents have constructed a two room house and you want that in the living room
there should be a provision of one electric bulb, one electric fan, a refrigerator and a plug point
for appliances of power upto 2 kilowatt. Draw a circuit diagram showing electric fuse and
earthing as safety devices.

29. Sahil took five solutions A, B, C, D and E and tested with universal indicator showed pH as 4,
1, 11, 7 and 9 respectively. Which solution is: (a) Neutral (b) Strongly alkaline (c) Strongly
acidic (d) Weakly acidic (E) Weakly alkaline?
Arrange the pH in increasing order of hydrogen ion concentration.

30. (a) Describe how a squirrel uses its hormonal system to react to a dangerous situation.
(b) How do sensory and motor neurons differ from one another?

31. Identify the acid and base which form sodium hydrogen carbonate. Write chemical equation in
support of your answer. State whether this compound is acidic, basic or neutral. Also, write its
pH value.

32. (a) What is an ecosystem? List its two main components.


(b) ‘The number of trophic levels in a food chain is limited’. Justify the statement.

33. Trace the sequence of events which occur when a bright light is focused on your eyes.

Prepared by: M. S. KumarSwamy, TGT(Maths) Page - 5 -


OR
List in tabular form three distinguishing features between autotrophic nutrition and
heterotrophic nutrition.

SECTION – D
Questions 34 to 36 carry 5 marks each.
34. A student wants to project the image of a candle flame on a screen 60 cm in front of a mirror by
keeping the candle flame at a distance of 15 cm from its pole.
(a) Which type of mirror should the student use?
(b) Find the magnification of the image produced.
(c) Find the distance between the object and its image.
(d) Draw a ray diagram to show the image formation in this case and mark the distance between
the object and its image.
OR
(a) Name the lens which can be used as a magnifying glass. For which position of the object a
convex lens form: (i) a real and inverted image of the same size as that of the object? (ii) a
virtual and erect image? Draw ray diagram to justify your answer in each case.
(b) One half of a convex lens is covered with a black paper. Will this lens produce a complete
image of the object? Draw ray diagram to justify your answer.

35. Why are certain compounds called hydrocarbons? Write the general formula for homologous
series of alkanes, alkenes and alkynes and also draw the structure of the first member of each
series. Write the name of the reaction that converts alkenes into alkanes and also write a
chemical equation to show the necessary conditions for the reaction to occur.
OR
(a) Explain why carbon forms covalent bond ? Give two reasons for carbon forming a large
number of compounds.
(b) Explain the formation of ammonia molecule.

36. Name three different glands associated with the structures labelled in digestive system as shown
in figure. Also write their secretions and their functions.

OR
(a) Define excretion.
(b) Name the basic filtration unit present in the kidney.
(c) Draw excretory system in human beings and label the following organs of excretory system
which perform following functions: (i) form urine. (ii) is a long tube which collects urine from
kidney. (iii) store urine until it is passed out.

Prepared by: M. S. KumarSwamy, TGT(Maths) Page - 6 -


SECTION – E (Case Study Based Questions)
Questions 35 to 37 carry 4 marks each.
37. Case Study – 1
Bacteria follow an asexual mode of reproduction, called binary fission. A single bacterium
divides into two daughter cells. These are identical to the parent cell as well as to each other.
Replication of DNA within parent bacterium marks the beginning of the fission. Eventually, cell
elongates to form two daughter cells.
The diagram shows the process of binary fission in bacteria

The rate and timing of reproduction depend upon the conditions like temperature and
availability of nutrients. When there is a favorable condition, E. coli or Escherichia coli
produces about 2 million bacteria every 7 hours.

(a) (i) What is the process of the division of a cell into several cells during reproduction in
Plasmodium?

(ii) A Planaria worm is cut horizontally in the middle into two halves P and Q such that the part
P contains the whole head of the worm. Another Planaria worm is cut vertically into two halves
R and S in such a way that both the cut pieces R and S contain half head each. Which of the cut
pieces of the two Planaria worms could regenerate to form the complete respective worms?

(b) The rapid spreading of bread would on slices of bread is due to spore formation. Explain
spore formation.
OR

(b) Suppose a bacterium reproduces by binary fission every 20 minutes. The new cells survive
and reproduce at the same rate. The graph below shows how the bacterial population would
grow from a single bacterium. What do you conclude?

Prepared by: M. S. KumarSwamy, TGT(Maths) Page - 7 -


38. Case Study – 2
Manoj performed an experiment to understand that heat is produced when a few drops of
concentrated sulphuric acid is slowly added into a beaker containing water. For this, he took 10
mL water in a beaker and added a few drops of concentrated H2SO4 to it. Then, he swirled the
beaker slowly. During the process, a vigourous reaction takes place. It is an exothermic process.

(a) Why is it recommended that the acid should be added to water and not water to the acid?
(b) How will the concentration of hydrogen ions gets affected if an acid is diluted?
(c) What is this process called? Define the process.
OR
If we have hydrochloric acid and acetic acid of equal concentration, which will be a stronger
acid and why?

39. Case Study – 3


A student wants to project the image of a candle flame on the walls of the school laboratory by
using a mirror.
(a) Which type of mirror should he use and why?
(b) At what distance, in terms of focal length of the mirror, should he place the candle flame to
get the magnified image on the wall?
(c) Draw a ray diagram to show the formation of the image in this case.
OR
(d) (i) To get the diminished image of the candle flame, where the object must be placed?
(ii) If the image formed by this mirror is inverted and real, then what will be its magnification?

Prepared by: M. S. KumarSwamy, TGT(Maths) Page - 8 -


KENDRIYA VIDYALAYA GACHIBOWLI, GPRA CAMPUS, HYD-32
SAMPLE PAPER TEST 02 FOR BOARD EXAM 2023

SUBJECT: SCIENCE MAX. MARKS: 80


CLASS : X DURATION: 3 HRS
General Instruction:
1. This Question Paper has 5 Sections A-E.
2. Section A has 20 MCQs carrying 1 mark each.
3. Section B has 5 questions carrying 02 marks each.
4. Section C has 6 questions carrying 03 marks each.
5. Section D has 4 questions carrying 05 marks each.
6. Section E has 3 case based integrated units of assessment (04 marks each) with sub-parts of the
values of 1, 1 and 2 marks each respectively.
7. All Questions are compulsory. However, an internal choice in 2 Qs of 5 marks, 2 Qs of 3 marks
and 2 Questions of 2 marks has been provided. An internal choice has been provided in the
2marks questions of Section E
8. Draw neat figures wherever required. Take π =22/7 wherever required if not stated.
SECTION – A
Questions 1 to 20 carry 1 mark each.
1. Which of the following a balanced equation for the following chemical reaction:

(a) Zn + Ag(NO3)2 → Zn(NO3)2 + 2Ag


(b) 2Zn + 2AgNO3 → Zn(NO3)2 + 2Ag
(c) Zn + 2AgNO3 → Zn(NO3)2 + 2Ag
(d) Zn + AgNO3 → Zn(NO3)2 + Ag

2. Which of the following will turn phenolphthalein pink?


(a) NaOH(aq) (b) HCl(aq) (c) CH3COOH(aq) (d) H2O

3. While studying the saponification reaction, what do you observe when you mix an equal amount
of colourless vegetable oil and 20% aqueous solution of NaOH in a beaker?
(a) The colour of the mixture has become dark brown.
(b) A brisk effervescence is taking place in the beaker.
(c) The outer surface of the beaker has become hot.
(d) The outer surface of the beaker has become cold.

4. Why do we store silver chloride in dark-coloured bottles?


(a) To prevent precipitation of silver chloride
(b) To prevent decomposition of silver chloride
(c) To promote decomposition of silver chloride
(d) All of these

Prepared by: M. S. KumarSwamy, TGT(Maths) Page - 1 -


5. Which of the following gives the correct increasing order of the atomic radii of O, F and N ?
(a) O, F, N (b) N, F, O (c) O, N, F (d) F, O, N
6. Identify X, Y and Z based on the basis of given information. X is a non-metal, which is an
important constituent of our food. It forms two oxides Y and Z. Y is toxic and it causes
suffocation and sometimes death. Z is responsible for global warming.
(a) X= C, Y= CO, Z=CO2 (b) X = S, Y= SO2, Z= SO3
(c) X=P, Y=P2O3, Z=P2O3 (d) X= O, Y = O2, Z = O3
7. Structural formulae of X and Y, which have the same molecular formula, C3H60 are :
(a) X = CH3CH2CHO, Y= CH3COCH3
(b) X = CH3OCH2CH3, Y= CH3COCH3
(c) X = CH3COCH3, Y = CH3COOH
(d) X= CH3CH2OCH3, Y = CH3CH2COOH
8. The correct sequence of reproductive stages seen in flowering plants is :
(a) Gametes, zygote, embryo, seedling
(b) Zygote, gametes, embryo, seedling
(c) Seedling, embryo, zygote, gametes
(d) Gametes, embryo, zygote, seedling

9. Mohan was confused and did not know how to draw the correct figure for geotropism. So, he
has drawn three figures, as shown below.

Which appears more accurate and why?


(a) (I), (II) (b) (I) only (c) (II), (III) (d) (II) only

10. Choose the correct statement about heart.


(a) The upper two chambers of human heart are called ventricles.
(b) The lower two chambers of human heart are called atrium.
(c) The chambers of the heart are separated by a partition called pericardium.
(d) The pulmonary artery carries deoxygenated blood from the right ventricle to the lungs.
11. Choose the correct heading for the given below points

I II
(i) It involves two individuals (i) It involves only one individual parent
(ii) It is a complex process (ii) It is a simple process
(iii) It produces genetic variation. (iii) It lacks genetic variations.

I II
(a) Asexual reproduction Sexual reproduction
(b) Asexual reproduction Vegetative reproduction
(c) Sexual reproduction Asexual reproduction
12. Four students plotted the sketch of the patterns of magnetic field lines representing the magnetic
field around a current carrying straight wire as shown in figures P, Q, R and S. Which one of the
following sketches is correct?

Prepared by: M. S. KumarSwamy, TGT(Maths) Page - 2 -


(a) P (b) Q (c) R (d) S

13. A pea plant is represented by Rr. This represents the


(a) Genetic composition of an individual (b) Characteristics which are visible in an organism.
(c) Alternate form of genes (d) None of these

14. The voltage-current (V-I) graph of a metallic conductor at two different temperatures T1 and T2
is shown:

At which temperature is the resistance higher?


(a) T2 (b) T1 (c) Equal at both T2 and T1 (d) Cannot say
15. As per Michael Faraday, the forefinger, middle finger and thumb indicate the direction of:
(a) magnetic field, force and current respectively.
(b) magnetic field, current and force respectively.
(c) current, force and magnetic field respectively.
(d) force, magnetic field and current respectively.
16. An electron enters a magnetic field at right angles to it as shown in fig.
The direction of the force acting on the electron will be:

(a) to the right (b) to the left (c) out of the page (d) into the page
Prepared by: M. S. KumarSwamy, TGT(Maths) Page - 3 -
DIRECTION: In the question number 17 and 20, a statement of Assertion (A) is followed by a
statement of Reason (R).
Choose the correct option
(a) Both assertion (A) and reason (R) are true and reason (R) is the correct explanation of
assertion (A)
(b) Both assertion (A) and reason (R) are true and reason (R) is not the correct explanation of
assertion (A)
(c) Assertion (A) is true but reason (R) is false.
(d) Assertion (A) is false but reason (R) is true.
Ans: (b) Both assertion (A) and reason (R) are true and reason (R) is not the correct explanation
of assertion (A)

17. Assertion (A): Fresh milk in which baking soda is added, takes a longer time to set as curd.
Reason (R): Baking soda decreases the pH value of fresh milk to below 6.

18. Assertion (a): The opening and closing of the pore is a function of the guard cells.
Reason (R): Stomatal pores are the site for exchange of gases by diffusion.

19. Assertion (A): Lipase help in emulsification of fats.


Reason (R): Lipase hydrolyses fats and oils.

20. Assertion (a): In a series circuit, the current is constant throughout the electric circuit.
Reason (R): All electric devices need equal currents to operate properly.

SECTION – B
Questions 21 to 25 carry 2 marks each.

21. Observe the given figure: What happens when the tube is heated?

(a) Write a balanced chemical equation of the reaction.


(b) Identify the brown gas X evolved.
22. Why does carbon become stable after sharing four electrons? What type of bond is formed by
sharing?
OR
Why are covalent compounds being poor conductors of electricity? Why do covalent
compounds have low melting and boiling points?
23. In birds and mammals, the left and right side of the heart are separated. Give reasons.
24. Neha did not want to have a child. So, she went to a doctor who suggested her to adopt
contraceptive methods. What are the reasons for adopting contraceptive methods?
25. Define the term power of accommodation. Write the modification in the curvature of the eye
lens which enables us to see the nearby objects clearly? Give relation between power and focal
length.
OR
Why is the sun visible to us 2 minutes before actual sunrise and 2 minutes after actual sunset?

Prepared by: M. S. KumarSwamy, TGT(Maths) Page - 4 -


26. DDT was sprayed in a lake to regulate breeding of mosquitoes. How would it affect the trophic
levels in the following food chain associated with a lake? Justify your answer.

SECTION – C
Questions 27 to 33 carry 3 marks each.
27. Based on the group valency of elements, write the molecular formula of the following
compounds giving justification for each:
(a) Oxides of first group elements.
(b) Halides of the elements of group 13.
(c) Compounds formed when an element A of group 2 combines with an element B of group 17.

28. Can two people with brown eyes have a blue-eyed baby? Explain.

29. Mention any three-information given by a chemical equation.


OR
State the law of conservation of mass as applicable in a chemical reaction. Illustrate with an
example.

30. Derive an expression for electric energy consumed in a device in terms of V, I and t, where V is
the potential difference applied to it, I is the current drawn by it and t is the time for which the
current flows ?

31. Draw ray diagrams showing the image formation by a convex mirror when an object is placed:
(a) at infinity (b) at finite distance from the mirror.

32. (a) Determine whether the P and Q-marked poles in the following diagram represent the North
or South pole. Why did you choose that response?
(b) Imagine that you are sitting in a chamber with your back to one wall. An electron beam,
moving horizontally from back wall towards the front wall, is deflected by a strong magnetic
field to your right side. What is the direction of magnetic field?
OR
State the rule to determine the direction of a (a) magnetic field produced around a straight
conductor-carrying current and (b) force experienced by a current-carrying straight conductor
placed in a magnetic field which is perpendicular to it.

33. (a) How does food chain differ from a food web?
(b) Make food chains in (i) forest (ii) pond.

SECTION – D
Questions 34 to 36 carry 5 marks each.

34. If, in Figure Rl = 10 Ohms, R2 = 40 Ohms, R, = 30 Ohms, R4 = 20 Ohms, RA = 60 Ohms and a


12 volt battery is connected to the arrangement, calculate: (a) the total resistance and (b) the
total current flowing in the circuit.

Prepared by: M. S. KumarSwamy, TGT(Maths) Page - 5 -


35. (a) An organic compound X with a molecular formula C undergoes oxidation in presence of
alkaline KMnO4 to form a compound Y. X on heating in presence of conc. H2SO4 at 443 K
gives Z, which on reaction gives back 'X'. Identify X, Y and Z and write the reactions involved.
(b) With hard water, "A" compound functions well. The production of shampoos and other
cleaning products uses it. A causes water pollution and isn’t entirely biodegradable. Hard water
makes 'B' less effective. There is no water pollution because it is 100 percent biodegradable.
Identify ‘A’ and ‘B’.
(c) A cyclic compound 'X' has molecular formula. It is unsaturated and burns with sooty flame.
Identify 'X' and write its structural formula. Will it decolorize bromine water or not and why?
(d) An organic compound 'A' is a constituent of antifreeze and has the molecular formula
C2H6O. Upon reaction with alkaline KMNO4 the compound 'A' is oxidized to another 'B’
identify the compound A' and 'B'. Write the chemical equation for the reaction which leads to
the formulation of 'B'.

36. What is sexual reproduction? Explain how this mode of reproduction gives rise to more viable
variations than asexual reproduction. How does this affect the evolution?
OR
(a) What are dominant and recessive traits?
(b) "Is it possible that a trait is inherited but may not be expressed in the next generation?" Give
a suitable example to justify this statement.

SECTION – E(Case Study Based Questions)


Questions 37 to 39 carry 4 marks each.
37. Case Study – 1
Compounds containing oxygen are of great interest in the field of chemistry. Because of
oxygen's high reactivity, it is most often found in compounds. Oxygen reacts rapidly with group
1 elements. All alkali metal oxides form basic solutions when dissolved in water. The principal
combustion product is the most stable product with respect to the reactants. For example, with
careful control of oxygen, the oxide M2O (where M represents any alkali metal) can be formed
with any of the alkali metals.
When a metal reacts with oxygen to form a metal oxide, a redox reaction occurs.
Metal + Oxygen → Metal oxide
Different metals burn in oxygen with different rates depending on their differing activeness.
The more reactive metal towards oxygen, the brighter and faster the combustion of the metal.
Given figure below shows the experiment is conducted to build the reactivity series of metals.
Oxygen that is used in combustion of other metals is provided by heating solid potassium
manganate (VII). When heated, lithium, sodium, potassium, rubidium, and cesium ignite
through combustion reactions with oxygen. Glass wool Metal filing Heat up Heat up Potassium
Manganate (VII) crystal

Prepared by: M. S. KumarSwamy, TGT(Maths) Page - 6 -


(a) Which metals do not react with oxygen even at high temperature?
(b) What happens when copper metal is heated in air?
(c) Almost all metals combine with oxygen to form metal oxides. Is this statement true?
OR
(c) Which metal oxides are soluble in water? Explain.

38. Case Study – 2


The growing size of the human population is a cause of concern for all people. The rate of birth
and death in a given population will determine its size. The human population growth year wise
is shown in the below figure. Reproduction is the process by which organisms increase their
population. The process of sexual maturation for reproduction is gradual and takes place while
general body growth is still going on. Some degree of sexual maturation does not necessarily
mean that the mind or body is ready for sexual act or for having and bringing up children.
Various contraceptive devices are being used by human beings to control the size of population.

(a) List two common signs of sexual maturation in boys and girls.
(b) What is the result of reckless female foeticide?
(c) Which contraceptive method changes the hormonal balance of the body? Give an example of
it
OR
Write two factors that determine the size of a population.

39. Case Study - 3


Dispersion of light occurs when white light is separated into its different constituent colors
because of refraction and Snell's law.
From Snell's law it can be seen that the angle of refraction of light in a prism depends on the
refractive index of the prism material.

Prepared by: M. S. KumarSwamy, TGT(Maths) Page - 7 -


Color l(nm) Freq. (Hz)
Red 760–647 4.3 × 1014
Orange 647-585 4.3 × 1014
Yellow 585-575 5.2 × 1014
Green 575-491 5.6 × 1014
Blue 491-424 6.6 × 1014

Since the refractive index varies with wavelength, the angle that the light is refracted by will
also vary with wavelength, causing an angular separation of the colors known as angular
dispersion.
For visible light, refraction indices n of most transparent materials (e.g., air, glasses) decrease
with increasing wavelength l:
Colour Wavelength Crown Flint
(nm) glass glass
Violet 396.9 1.533 1.663
Blue 486.1 1.523 1.639
Yellow 589.3 1.517 1.627
Red 656.3 1.515 1.622

Most often seen in recently made puddles on the sides of roads, the oil refracts light much the
same way a rainbow does. Simply put, the thin layer of oil floating on top of the water refracts
the light which then bounces back up off the water underneath, splitting the light rays creating a
pool of rainbow colours.
(a) Which ray is least deviated by a prism?
(b) Which colour of light which has the minimum velocity in the glass prism?
(c) Which optical phenomenon is involved in formation of rainbow?
OR
(c) What is the angle of deviation (d) of a prism?

Prepared by: M. S. KumarSwamy, TGT(Maths) Page - 8 -


KENDRIYA VIDYALAYA GACHIBOWLI, GPRA CAMPUS, HYD-32
SAMPLE PAPER TEST 03 FOR BOARD EXAM 2023

SUBJECT: SCIENCE MAX. MARKS: 80


CLASS : X DURATION: 3 HRS
General Instruction:
1. This Question Paper has 5 Sections A-E.
2. Section A has 20 MCQs carrying 1 mark each.
3. Section B has 6 questions carrying 02 marks each.
4. Section C has 6 questions carrying 03 marks each.
5. Section D has 4 questions carrying 05 marks each.
6. Section E has 3 case based integrated units of assessment (04 marks each) with sub-parts of the
values of 1, 1 and 2 marks each respectively.
7. All Questions are compulsory. However, an internal choice in 2 Qs of 5 marks, 2 Qs of 3 marks
and 2 Questions of 2 marks has been provided. An internal choice has been provided in the
2marks questions of Section E
8. Draw neat figures wherever required. Take π =22/7 wherever required if not stated.
SECTION – A
Questions 1 to 20 carry 1 mark each.
1. In an attempt to demonstrate electrical conductivity through an electrolyte, the following
apparatus (see below Figure) was set up.

Which among the following statement(s) is(are) correct?


(i) Bulb will not glow because electrolyte is not acidic
(ii) Bulb will glow because NaOH is a strong base and furnishes ions for conduction.
(iii) Bulb will not glow because circuit is incomplete
(iv) Bulb will not glow because it depends upon the type of electrolytic solution
(a) (i) and (iii) (b) (ii) and (iv) (c) (ii) only (d) (iv) only

2. Why are ionic compounds hard crystalline solids?


(a) Due to the strong force of attraction between positive ions.
(b) Due to the strong force of attraction between negative ions.
(c) Both (a) and (b)
(d) Due to the strong force of attraction between molecules.

3. Which among the following statement(s) is (are) true? Exposure of silver chloride to sunlight
for a long duration turns grey due to
(i) the formation of silver by decomposition of silver chloride
(ii) sublimation of silver chloride
(iii) decomposition of chlorine gas from silver chloride
(iv) oxidation of silver chloride
(a) (i) only (b) (i) and (iii) (c) (ii) and (iii) (d) (iv) only

Prepared by: M. S. KumarSwamy, TGT(Maths) Page - 1 -


4. Equal volumes of hydrochloric acid and sodium hydroxide solutions of same concentration are
mixed and the pH of the resulting solution is checked with a pH paper. What would be the
colour obtained? (You may use colour guide given in below Figure)

(a) Red (b) Yellow (c) Yellowish green (d) Blue

5. If the key (K) in the arrangement (figure) is taken out (i.e., circuit is made open) and the
magnetic field lines are drawn over the horizontal plane ABCD, the lines are

(a) concentric circles (b) elliptical in shapes (c) straight lines parallel to each other
(d) concentric circles near the point O, but of elliptical shapes as we go away from it

6. Listed here is the reactivity of certain metals.


Metal Reaction with air Reaction with water Reaction with dilute
acids
Gold Does not oxidise or No reaction No reaction
burn
Sodium Burns vigorously to Violent reaction Violent reaction
form oxide
Zinc Burns to form oxides Reacts on heating Reacts to produce
with water hydrogen
Platinum Does not oxidise or No reaction No reaction
burn
Which of the above metals are likely to be obtained in their pure states from the Earth’s crust?
(a) Gold only (b) Sodium only (c) Gold and platinum (d) Zinc and sodium

7. The image shows a bud developing on a Hydra.

Prepared by: M. S. KumarSwamy, TGT(Maths) Page - 2 -


How does the bud develop in the Hydra?
(a) bud develops due to separation of body parts of Hydra
(b) bud develops due to repetitive cell division at a specific site
(c) bud develops due to change in the environmental conditions
(d) develops due to attachment of another Hydra at a specific site

8. A student studies that acetic acid is a saturated compound.


The structure of the compound is shown.

Why is acetic acid classified as a saturated compound?


(a) Because there is a single bond between the carbon atoms.
(b) Because there is a double bond between the carbon and oxygen atoms.
(c) Because there is a single bond between the carbon and hydrogen atoms.
(d) Because there is a single bond between the carbon and hydroxide diatom.

9. Which is the correct sequence of parts in human alimentary canal?


(a) Mouth → stomach → small intestine → oesophagus → large intestine
(b) Mouth → oesophagus → stomach → large intestine → small intestine
(c) Mouth → stomach → oesophagus → small intestine → large intestine
(d) Mouth → oesophagus → stomach → small intestine → large intestine

10. The opening and closing of the stomatal pore depends upon:
(a) oxygen (b) temperature (c) water in guard cells (d) concentration of CO2 in stomata

11. What is the direction of magnetic field at a point A above the wire carrying current I as shown
in figure?

(a) Out of the page (b) Into the page (c) Up the page (d) Down the page

12. Two pea plants one with round green seeds (RRyy) and another with wrinkled yellow (rrYY)
seeds produce F1 progeny that have round yellow (RrYy) seeds. When F1 plants are self-
pollinated, the F2 progeny will have new combination of characters. Choose the new
combination from the following
(i) Round, yellow (ii) Round, green (iii) Wrinkled, yellow (iv) Wrinkled, green
(a) (i) and (ii) (b) (ii) and (iii) (c) (i) and (iv) (d) (i) and (iii)

13. Involuntary actions in the body are controlled by


(a) medulla in fore brain (b) medulla in mid brain
(c) medulla in hind brain (d) medulla in spinal cord
14. A metal carbonate reacts with a solution X which forms a salt, water, and a gas Y. What are X
and Y?
(a) X: sodium hydroxide; Y: carbon dioxide
(b) X: sodium hydroxide; Y: hydrogen
(c) X: hydrochloric acid; Y: carbon dioxide
(d) X: hydrochloric acid; Y: hydrogen

Prepared by: M. S. KumarSwamy, TGT(Maths) Page - 3 -


15. A student connected a simple circuit for verifying Ohm’s law.

(I) Reading of an ammeter connected in the circuit becomes half when resistance is doubled.
(II) Reading of an ammeter connected in the circuit becomes double when resistance is halved.
(III) Reading of an ammeter connected in the circuit becomes half when voltage is doubled.
(IV) Reading of an ammeter connected in the circuit becomes half when voltage is halved.
Select the incorrect statement:
(a) Only (I) (b) Only (II) (c) Only (III) (d) Both (II) and (III) (1)

16. In order to reduce electricity consumption at home, what kind of appliance should one
purchase?
(a) one which draws low power (b) one which produces less heat
(c) one which operates at a higher voltage (d) one which draws a high amount of current

DIRECTION: In the question number 17 and 20, a statement of Assertion (A) is followed by a
statement of Reason (R).
Choose the correct option
(a) Both assertion (A) and reason (R) are true and reason (R) is the correct explanation of
assertion (A)
(b) Both assertion (A) and reason (R) are true and reason (R) is not the correct explanation of
assertion (A)
(c) Assertion (A) is true but reason (R) is false.
(d) Assertion (A) is false but reason (R) is true.
Ans: (b) Both assertion (A) and reason (R) are true and reason (R) is not the correct explanation
of assertion (A)

17. Assertion (A): The reaction MnO2 + 4HCl → MnCl2 + 2H2O + Cl2 is an example of a redox
reaction.
Reason (R): In this reaction, HCl is reduced to Cl2 whereas MnO2 is oxidised to MnCl2.

18. Assertion (A): Carbohydrate digestion mainly takes place in small intestine.
Reason (R): Pancreatic juice contains the enzyme lactase.

19. Assertion (A): A geneticist crossed two pea plants and got 50% tall and 50% dwarf in the
progeny.
Reason (R): One plant was heterozygous tall and the other was dwarf.

20. Assertion (A): Alternating Current is used in household supply.


Reason (R): AC electric power can be transmitted over long distances without much loss of
energy.

SECTION – B
Questions 21 to 26 carry 2 marks each.

21. A substance X used for coating iron articles is added to a blue solution of a reddish brown metal
Y, the color of the solution gets discharged Identify X and Y & also the type of reaction.

Prepared by: M. S. KumarSwamy, TGT(Maths) Page - 4 -


22. The number of malarial patients in a village increased tremendously when large number of frogs
were exported from the village. What could be the cause for this?

23. Why did Mendel choose the pea plant for his experiments?

24. (a) Identify the organism which causes Kala-azar. How does this organism reproduce?
(b) Draw a diagram showing its reproduction.

25. What is peptic ulcer? How is peptic ulcer caused?


OR
List two different functions performed by pancreas in our body.

26. On what factor does the colour of scattered light depend? Explain with an example.
OR
What would happen if danger lights were blue in colour? Justify your answer.

SECTION – C
Questions 27 to 33 carry 3 marks each.
27. Identify the type of chemical reaction in the following statements and define each of them
(a) Digestion of food in our body
(b) Rusting of iron
(c) Heating of manganese dioxide with aluminium powder

28. Draw the pattern of magnetic field lines around a current carrying solenoid and mark the North
and South poles. What can you say about the magnetic field inside the solenoid? How can a
solenoid be used to form an electromagnet?
OR
(i) Draw the pattern of magnetic field lines due to a magnetic field through and around a current
carrying circular loop.
(ii) Name and state the rule to find out the direction of magnetic field inside and around the
loop.

29. In the following schematic diagram for the preparation of hydrogen gas as shown in below
Figure, what would happen if following changes are made?

(a) In place of zinc granules, same amount of zinc dust is taken in the test tube
(b) Instead of dilute sulphuric acid, dilute hydrochloric acid is taken
(c) In place of zinc, copper turnings are taken
(d) Sodium hydroxide is taken in place of dilute sulphuric acid and the tube is heated.

Prepared by: M. S. KumarSwamy, TGT(Maths) Page - 5 -


30. (a) What are the functions of kidneys?
(b) Name the filtration units present in kidneys.
(c) Name two substances which are selectively reabsorbed from nephric filtrate into the blood.
OR
Explain the process of breathing in man

31. A student holding a mirror in his hand, directed the reflecting surface of the mirror towards the
Sun. He then directed the reflected light on to a sheet of paper held close to the mirror.
(i) What should he do to burn the paper?
(ii) Which type of mirror does he have?
(iii) Will he be able to determine the approximate value of focal length of this mirror from this
activity? Give reason and draw ray diagram to justify your answer in this case.

32. ‘‘A lens can form a magnified erect image as well as magnified inverted image of an object
placed in front of it.’’ State the nature of this lens and draw ray diagrams to justify the above
statement. Mark the positions of O, F and 2F in the diagram.

33. Ozone therapy is a controversial alternative medical practice that uses ozone gas to fight
disease. Aditya asked his mother whether this ozone therapy is legal to be used. His mother
explained that in 2019, the Food and Drug Administation (FDA) warned against using this
therapy because there is not enough evidence to conclude that it is safe for medical use. They
say that it has no known useful application in supportive or preventive medicine. It is a toxic
and harmful air pollution on ground level. Why is damage to the ozone layer is a cause for
concern? What steps are being taken to limit this damage?

SECTION – D
Questions 34 to 36 carry 5 marks each.

34. Sahil bought a compound ' X' on electrolysis in aqueous solution produces a strong base Y'
along with two gases 'A' and 'B'. 'B' is used in manufacture of bleaching powder. Identify X, Y,
A and B. Write chemical equations
OR
Raman took a sodium compound 'X', which is also used in soda-acid fire extinguisher, and
heated it gives a sodium compound 'Y' along with water and carbon dioxide. 'Y' on
crystallisation forms a compound 'Z'.
(a) Identify 'X', 'Y' and 'Z'. Write chemical equations of the reactions taking place.
(b) How can we obtain Y from Z? Write equation.
(c) Write any two uses of the compound 'Z'.

35. (i) For the combination of resistors shown in the following figure, find the equivalent resistance
between M & N.

(ii) State Joule’s law of heating.


(iii) Why we need a 5 A fuse for an electric iron which consumes 1 kW power at 220 V?
(iv) Why is it impracticable to connect an electric bulb and an electric heater in series?

Prepared by: M. S. KumarSwamy, TGT(Maths) Page - 6 -


36. (i) Write the functions of each of the following parts in a human female reproductive system: (a)
Ovary (b) Uterus (c) Fallopian tube
(ii) Write the structure and functions of placenta in a human female.
OR
(i) “Use of a condom is beneficial for both the sexes involved in a sexual act.” Justify this
statement giving two reasons.
(ii) How do oral contraceptive help in avoiding pregnancies?
(iii) What is sex selective abortion? How does it affect a healthy society? (State any one
consequence)

SECTION – E(Case Study Based Questions)


Questions 37 to 39 carry 4 marks each.
37. Case Study – 1
The nature of non-metals is strongly electronegative. To obtain the nearest noble gas
configuration, they frequently add electrons to their valence shell. They become anions as a
result, which makes them effective oxidising agents.

X + ne- → Xn-
(non-metal atom) (anion)

They react with air or oxygen on heating to form oxides which react with water to form acids.
Thus, non-metal oxides are acidic in nature. Non-metals do not react with dilute acids at all.
This is because they are electronegative and therefore, cannot displace hydrogen from acids but
they form covalent hydrides when heated with hydrogen.
(a) Name the acid formed when sulphur trioxide reacts with water.
(b) An element 'X' forms an oxide XO, which is a very useful gas used in the process of
photosynthesis. Identify the element 'X'.
(c) Non-metals generally act as oxidising agents. Justify. Identify an element which produces
basic oxide on reacting with oxygen?
OR
(c) Name three elements which form covalent hydride?

38. Case Study – 2


In human beings, the sex of the individual is largely genetically determined. When two germ
cells combine, they will restore the normal number of chromosomes in the progeny, ensuring
the stability of the DNA of the species. Nivedita has dark brown eyes, like her mother. But she
has the same shaped nose as her father.
(i) Explain why Nivedita has features from both of her parents.
(ii) Explain why Nivedita is genetically female.
OR
(ii) In humans, the gene for black hair colour is B and gene for brown hair colour is b. What will
be the hair colour of person having the genetic constitution (a) BB (b) bb?

39. Case Study – 3


A spherical mirror is a mirror which has the shape of a piece cut out of a spherical surface.
There are two types of spherical mirrors: concave, and convex. As is well-known, these types of
mirrors magnify objects placed close to them. It has been observed that as rays from a distant
object depart further from the principal axis of a concave mirror they are brought to a focus ever
closer to the mirror, as shown in fig. below: This lack of perfect focusing of a spherical mirror is
called spherical aberration.

Prepared by: M. S. KumarSwamy, TGT(Maths) Page - 7 -


(a) Where is the principal focus of a parabolic mirror?
(b) After reflection from a concave mirror, rays of light parallel. to the principal. axis converge
at a point. What do we call that point?
OR
(b) Define centre of curvature and focal length of a mirror.

Prepared by: M. S. KumarSwamy, TGT(Maths) Page - 8 -


KENDRIYA VIDYALAYA GACHIBOWLI, GPRA CAMPUS, HYD-32
SAMPLE PAPER TEST 04 FOR BOARD EXAM 2023

SUBJECT: SCIENCE MAX. MARKS: 80


CLASS : X DURATION: 3 HRS
General Instruction:
1. This Question Paper has 5 Sections A-E.
2. Section A has 20 MCQs carrying 1 mark each.
3. Section B has 5 questions carrying 02 marks each.
4. Section C has 6 questions carrying 03 marks each.
5. Section D has 4 questions carrying 05 marks each.
6. Section E has 3 case based integrated units of assessment (04 marks each) with sub-parts of the
values of 1, 1 and 2 marks each respectively.
7. All Questions are compulsory. However, an internal choice in 2 Qs of 5 marks, 2 Qs of 3 marks
and 2 Questions of 2 marks has been provided. An internal choice has been provided in the
2marks questions of Section E
8. Draw neat figures wherever required. Take π =22/7 wherever required if not stated.
SECTION – A
Questions 1 to 20 carry 1 mark each.
1. Identify gas A in the following experiment.

(a) Nitrogen (b) Hydrogen (c) Oxygen (d) Carbon dioxide

2. A scientist in a chemistry lab wants to make salt of pH 5.5 using acid and base. The table shows
the acid and base present in the lab.
1 HCl
2 NaOH
3 H2CO3
4 NH4OH
5 CH3COOH
Which of the acid and base he should use for the reaction?
(a) CH3COOH and NaOH (b) HCl and NaOH (c) HCl and NH4OH (d) H2CO3 and NaOH

3. Which of the following are correct structural isomers of butane?

(a) (i) and (iii) (b) (ii) and (iv) (c) (i) and (ii) (d) (iii) and (iv)

Prepared by: M. S. KumarSwamy, TGT(Maths) Page - 1 -


4. The image shows the transport of gases in body through heart and lungs.

Which option correctly shows the transport of oxygen to the cell?


(a) Lungs → pulmonary vein → left atrium → left ventricle → aorta → body cells.
(b) Lungs → pulmonary artery → right atrium → right ventricle → vena cava → body cells.
(c) Lungs → pulmonary artery → left atrium → left ventricle → vena cava → body cells.
(d) Lungs → pulmonary vein → right atrium → right ventricle → aorta → body cells.

5. Which of the following occurs during oxygen shortage in muscle cells?

(a) Only X (b) Only Y (c) Only Z (d) Any of them - X, Y or Z

6. A cell, a resistor, a key and ammeter are arranged as shown in circuit diagrams.

The current recorded in the ammeter will be


(a) maximum in (i) (b) maximum in (ii) (c) maximum in (iii) (d) same in all three circuits

7. The path of a ray of light coming from air passing through a rectangular glass slab is traced by
four students shown as A, B, C and D in the figure. Which one of them is correct?

Prepared by: M. S. KumarSwamy, TGT(Maths) Page - 2 -


8. Which of the following statements about transmission of nerve impulse is incorrect?
(a) Nerve impulse travels from dendritic end towards axonal end
(b) At the dendritic end electrical impulses bring about the release of some chemicals which
generate an electrical impulse at the axonal end of another neuron
(c) The chemicals released from the axonal end of one neuron cross the synapse and generate a
similar electrical impulse in a dendrite of another neuron
(d) A neuron transmits electrical impulses not only to another neuron but also to muscle and
gland cells

9. A child is standing in front of a magic mirror. She finds the image of her head bigger, the
middle portion of her body of the same size and that of the legs smaller. The following is the
order of combinations for the magic mirror from the top.
(a) Plane, convex and concave (b) Convex, concave and plane
(c) Concave, plane and convex (d) Convex, plane and concave

10. If the structure marked X in the diagram given below is blocked, then which of the processes
will not occur?

(a) Transpiration and respiration


(b) Transpiration, photosynthesis and respiration
(c) Respiration, transpiration and transportation
(d) Respiration and photosynthesis

11. A student carries out an experiment and plots the V - I graph of three samples of nichrome wire
with resistances R1, R2 and R3 respectively as shown in figure. Which of the following is true?

(a) R1 = R2 = R3 (b) R1 > R2 > R3 (c) R3 > R2 > R1 (d) R2 > R3 > R1

12. Which option illustrates the location of centre that controls the feelings associated with hunger
(M) and the centre that allows a person to walk in a straight line (N)?

Prepared by: M. S. KumarSwamy, TGT(Maths) Page - 3 -


13. In the given food chain, suppose the amount of energy at the fourth trophic level is 5 kJ, what
will be the energy available at the producer level?
Grass → Grasshopper → Frog → Snake → Hawk
(a) 5 kJ (b) 50 kJ (c) 500 kJ (d) 5000 kJ

14. Disposable plastic plates should not be used because:


(a) they are made of materials with light weight
(b) they are made of toxic materials
(c) they are made of biodegradable materials
(d) they are made of non-biodegradable materials

15. Anand took four colourless solutions P, Q, R and S, and performed the following tests. What is
the definite conclusion that Anand can reach?
Solution P Solution Q Solution R Solution S
With methyl No change in Turns red No change in No change in
orange colour colour colour
With No change in No change in No change in Turns pink
phenolphthalein colour colour colour
With red litmus No change in No change in No change in Turns litmus
colour colour colour blue
With blue litmus No change in Turns litmus red No change in No change in
colour colour colour
(a) Both P and S are salt solutions. (b) Both Q and S are basic solutions.
(c) Both Q and R are salt solutions. (d) Both P and R are neutral solutions.

16. A cylindrical conductor of length l and uniform area of cross section A has resistance R.
Another conductor of length 2l and resistance R of the same material has area of crosssection:
(a) A/2 (b) 3A/2 (c) 2A (d) 3A

DIRECTION: In the question number 17 and 20, a statement of Assertion (A) is followed by a
statement of Reason (R).
Choose the correct option
(a) Both assertion (A) and reason (R) are true and reason (R) is the correct explanation of
assertion (A)
(b) Both assertion (A) and reason (R) are true and reason (R) is not the correct explanation of
assertion (A)
Prepared by: M. S. KumarSwamy, TGT(Maths) Page - 4 -
(c) Assertion (A) is true but reason (R) is false.
(d) Assertion (A) is false but reason (R) is true.

17. Assertion (A) : The energy of charged particle moving in a uniform magnetic field does not
change.
Reason (R) : Work done by magnetic field on the charge is zero.

18. Assertion (A) : The compass placed near the current-carrying wire remains stationary.
Reason (R) : The current flowing through a wire gives rise to a magnetic field.

19. Assertion (A) : A convex mirror is used as a rear view driver's mirror.
Reason (R) : Convex mirrors have a wider field of view as they are curved outwards. They also
give an erect, although diminished image.

20. Assertion (A) : Acquired traits cannot be passed from one generation to next generation.
Reason (R) : Inaccuracy during DNA copying of acquired trait is minimum.

SECTION – B
Questions 21 to 26 carry 2 marks each.

21. Why do fire flies glow at night?


OR
A dilute ferrous sulphate solution was gradually added to the beaker containing acidified
potassium permanganate solution. The light purple colour of the solution fades and finally
disappears. Write the correct explanation for this observation.

22. Give one example of an organic compound present in biogas. Draw electron dot structure of this
compound.

23. Ibrahim applied sodium hydroxide to the lustrous, divalent element M. In the reaction mixture,
he saw bubbles beginning to form. When hydrochloric acid was used to treat this element, he
recorded the same findings. Tell him how to recognise the gas that was produced. For both
reactions, write chemical equations.

24. ‘Variations are beneficial to the species but not necessarily for the individual’. Justify.

25. Draw a ray diagram to explain the term angle of deviation.


OR
Write about power of accommodation of human eye. Explain why the image distance in the eye
does not change when we change the distance of an object from the eye?

26. “All plants give out oxygen during day and carbon dioxide during night”. Do you agree with
this statement? Give reason.
SECTION – C
Questions 27 to 33 carry 3 marks each.
27. A chemical compound ‘X’ is used in the soap and glass industry. It is prepared from brine.
(i) Write the chemical name, common name and chemical formula of ‘X’.
(ii) Write the equation involved in its preparation.
(iii) What happens when it is treated with water containing Ca or Mg salts?

28. 1 g of copper powder was taken in a China dish and heated. What change takes place on
heating? When hydrogen gas is passed over this heated substance, a visible change is seen in it.
Give the chemical equations of reactions, the name and the color of the products formed in each
case.

Prepared by: M. S. KumarSwamy, TGT(Maths) Page - 5 -


29. Study the following circuit and answer the questions that follows:
(a) State the type of combination of two resistors in the circuit.
(b) How much current is flowing through (i) 10 Ohms and (ii) 15 Ohms resistor?
(c) What is the ammeter reading?

30. Rakesh performed the experiment to study the dispersion of light by a glass prism and drew the
following figure.
(a) The colour at position marked 3 and 5 are similar to the colour of the sky and the colour of
gold metal respectively. Is the above statement made by the student correct or incorrect? Justify.
(b) Which of the above shown positions correspond approximately to the colour of:
(i) a solution of potassium permanganate?
(ii) danger or stop signal lights?
OR

Refractive indices of media A, B, C and D are given:


Media: A B C D
Refractive Index: 133 144 152 165
In which of these four media is the speed of light () Minimum and (ji) Maximum?
Give reasons. Find the refractive index of medium A with respect to medium B.

31. A cross was carried out between pure breed tall pea plant with pure dwarf pea plant and F1
progeny was obtained. Later, F1, progeny was selfed to obtain F2 progeny. Answer the following
questions:
(a) What is the phenotype of the F1 progeny and why?
(b) Give the phenotypic ratio of the F2 progeny.
(c) Why is F2 progeny different from the F1 progeny?

32. Plastic cups were used to serve tea in trains in early days—these could be returned to the
vendors, cleaned and reused. Later, Kulhads were used instead of plastic cups. Now, paper cups
are used for serving tea. What are the reasons for the shift from Plastic to Kulhads and then
finally to paper cups?

33. State the events occurring during the process of photosynthesis. Is it essential that these steps
take place one after the other immediately?
OR
Bile juice does not have any digestive enzyme but still plays a significant role in the process of
digestion. Justify the statement.

SECTION – D
Questions 34 to 36 carry 5 marks each.

34. What are esters? How are esters prepared? Write the chemical equation for the reaction
involved. What happens when an ester reacts with sodium hydroxide? Write the chemical
equation for the reaction and also state the name and use of this reaction.
Prepared by: M. S. KumarSwamy, TGT(Maths) Page - 6 -
OR
A compound C (molecular formula, C2H4O2) reacts with Na-metal to form a compound R and
evolves a gas which burns with a pop sound. Compound C on treatment with an alcohol A in
presence of an acid forms a sweet smelling compound S (molecular formula C3H6O2). On
addition of NaOH to C, it also gives R and water. S on treatment with NaOH solution gives
back R and A.
Identify C, R, A, S and write down the reactions involved.

35. (i) Label the endocrine glands given in figure below:

(ii) A hot object has been touched by you. Draw a diagram that shows the steps that result in a
response, such as a hasty hand pullback.
(iii) How are involuntary actions and reflex actions different from each other?

36. What is a solenoid? Draw the pattern of magnetic field lines of (i) a current carrying solenoid
and (ii) a bar magnet. List two distinguishing features between the two fields.
OR
With the help of a labelled circuit diagram illustrating the pattern of field lines of the magnetic
field around a current-carrying straight long conducting wire. How is the right-hand thumb rule
useful to find the direction of the magnetic field associated with a current-carrying conductor?

SECTION – E(Case Study Based Questions)


Questions 37 to 39 carry 4 marks each.
37. Case Study – 1
We know that the characteristics of image formed by a concave mirror depend on the position of
the object with respect to the mirror.
When an object is placed between F and infinity, the image formed is real and inverted. But
when the object is placed between F and mirror it cannot be obtained on the screen. The image
formed in this case is virtual, erect and magnified. Such image may be seen by looking in the
mirror directly.

Prepared by: M. S. KumarSwamy, TGT(Maths) Page - 7 -


When the object is moved from focus towards infinity, the image moves from infinity towards
focus and its size decreases.
When object is placed at 2F image of the same size is formed at 2F, itself.
(i) What will be the nature of image if an object is placed 10 cm in front of a concave mirror of
focal length 20 cm?
(ii) What is the minimum distance between the object and its real image for concave mirror?
(iii) A candle flame 3 cm high is placed at a distance of 3 m from a wall. How far from the wall
must a concave mirror be placed in order that it may form an image of the flame 9 cm high on
the wall?
OR
(iii) Draw a ray diagram when an object is placed near a concave mirror at a distance of one-
fourth the radius of curvature of the concave mirror?

38. Case Study – 2


A student added 10 g of calcium carbonate in a rigid container, secured it tightly and started to
heat it. After some time, an increase in pressure was observed, the pressure reading was then
noted at intervals of 5 mins and plotted against time, in a graph as shown below.

(i) During which interval did maximum decomposition took place?


(ii) Marble statues are corroded or stained when they repeatedly come into contact with polluted
rain water. Identify the main reason.

(iii) What happens when calcium carbonate decompose? What could be done to increase the rate
of decomposition of CaCO3?
OR
(iii) Is decomposition of limestone endothermic? Give reason.
39. Case Study - 3
Have you ever wondered how water reaches the top of tall trees, or for that matter how and why
substances move from one cell to the other? Plants need to move molecules over very long
distances, much more than animals do; they also do not have a circulatory system in place.
Prepared by: M. S. KumarSwamy, TGT(Maths) Page - 8 -
Water taken up by the roots has to reach all parts of the plant, up to the very tip of the growing
stem. The photosynthates or food synthesised by the leaves have also to be moved to all parts
including the root tips embedded deep inside the soil. Movement across short distances, say
within the cell, across the membranes and from cell to cell within the tissue has also to take
place. Over small distances substances move by diffusion and by cytoplasmic streaming
supplemented by active transport. Transport over longer distances proceeds through the vascular
system (the xylem and the phloem) and is called translocation.

The table below gives the results of an experiment carried out to study the factors affecting the
rate of transpiration:

Amount of water transpired in 1 Hour (ml)


Normal With Fan With Heater With Lamp
Arrowhead 3.6 7.5 6.6 4
Coleus 0.9 6 3.9 3
Devil's Ivy 2.9 4.6 4.1 3
Dieffenbachia 4.1 7.7 6 3.9
English Ivy 1.8 5.1 3.2 2.1
Geranium 1.2 4.7 5.8 2.4
Rubber Plant 4.9 8.4 6.8 4.3
Weeping Fig 3.3 6.1 4.9 2.5
Zebra Plant 4.2 7.6 6.1 3.2

(a) What do we call the absorption and upward movement of minerals from roots to leaves.
(b) Root pressure is involved in transpiration. Is this statement true?
(c) Effect of root pressure in transport of water is more important at night. Justify.
OR
(c) What is the function of phloem?

Prepared by: M. S. KumarSwamy, TGT(Maths) Page - 9 -


KENDRIYA VIDYALAYA GACHIBOWLI, GPRA CAMPUS, HYD-32
SAMPLE PAPER TEST 05 FOR BOARD EXAM 2023

SUBJECT: SCIENCE MAX. MARKS: 80


CLASS : X DURATION: 3 HRS
General Instruction:
1. This Question Paper has 5 Sections A-E.
2. Section A has 20 MCQs carrying 1 mark each, Section B has 5 questions carrying 02 marks
each, Section C has 6 questions carrying 03 marks each, Section D has 4 questions carrying 05
marks each and Section E has 3 case based integrated units of assessment (04 marks each) with
sub-parts of the values of 1, 1 and 2 marks each respectively.
3. All Questions are compulsory. However, an internal choice in 2 Qs of 5 marks, 2 Qs of 3 marks
and 2 Questions of 2 marks has been provided. An internal choice has been provided in the 2
marks questions of Section E
SECTION – A
Questions 1 to 20 carry 1 mark each.
1. A student adds an equal amount of copper sulphate solution in two beakers. He adds zinc in
beaker P and silver in beaker Q. The student observes that the color of the solution in beaker P
changes while no change is observed in beaker Q. Which option arranges the metals in
increasing order of reactivity?
(a) Copper-silver-zinc (b) Zinc-copper-silver (c) Silver-copper-zinc (d) Silver-zinc-copper
2. Study the diagram given alongside and identify the gas formed in the reaction.

(a) Carbon dioxide which extinguishes the burning candle.


(b) Oxygen due to which the candle burns more brightly.
(c) Sulphur dioxide which produces a suffocating smell.
(d) Hydrogen which while burning produces a popping sound.
3. A sample of soil is mixed with water and allowed to settle. The clear supernatant solution turns
the pH paper yellowish-orange. Which of the following would change the colour of this pH
paper to greenish-blue?
(a) Lemon juice (b) Vinegar (c) Common salt (d) An antacid
4. Generally metals react with acids to give salt and hydrogen gas. Which of the following acids
does not give hydrogen gas on reacting with metals (except Mn and Mg)?
(a) H2SO4 (b) HCl (c) HNO3 (d) All of these
5. What happens when a solution of an acid is mixed with a solution of a base in a test tube?
(i) The temperature of the solution increases (ii) The temperature of the solution decreases
(iii) The temperature of the solution remains the same (iv) Salt formation takes place
(a) (i) only (b) (i) and (iii) (c) (ii) and (iii) (d) (i) and (iv)
6. Which of the following reactions is a neutralisation reaction?
(a) 4Na + O2 → 2Na2O (b) Fe + 2HCl → FeCl2 + H2
(c) MgO + H2O → Mg(OH)2 (d) HNO3 + NaOH → NaNO3 + H2O
Prepared by: M. S. KumarSwamy, TGT(Maths) Page - 1 -
7. Identify the unsaturated compounds from the following
(i) Propane (ii) Propene (iii) Propyne (iv) Chloropropane
(a) (i) and (ii) (b) (ii) and (iv) (c) (iii) and (iv) (d) (ii) and (iii)
8. Which one among the following is not removed as a waste product from the body of a plant?
(a) Resins and Gums (b) Urea (c) Dry Leaves (d) Excess Water
9. A student was asked to write a stepwise procedure to demonstrate that carbon dioxide is
necessary for photosynthesis. He wrote the following steps. The wrongly worded step is:

(a) Both potted plants are kept in dark room for at least three days.
(b) Bottom of the bell jars is sealed to make them air tight.
(c) Both potted plants are kept in sunlight after the starch test.
(d) A leaf from both the plants is taken to test the presence of starch.
10. Which of the following statement is incorrect?
(a) For every hormone there is a gene. (b) For every protein there is a gene.
(c) For production of every enzyme there is a gene. (d) For every molecule of fat there is a gene.
11. Select the mismatched pair
(a) Adrenaline: Pituitary gland (b) Testosterone: Testes
(c) Estrogen : Ovary (d) Thyroxin: Thyroid gland
12. Electrical impulse travels in a neuron from
(a) Dendrite → axon → axonal end → cell body
(b) Cell body → dendrite → axon → axonal end
(c) Dendrite → cell body → axon → axonal end
(d) Axonal end → axon → cell body → dendrite
13. Four cells each of emf 1.5V and the internal resistance 0.5 Ω are connected in series but one cell
is wrongly connected as shown in figure.

The net voltage and net internal resistance between A and B is


(a) 6 V, 2 Ω (b) 4.5 V, 1.5 Ω (c) 3V, 1 Ω (d) 3V, 2 Ω
14. What is the direction of magnetic field at a point A above the wire carrying current I as shown
in figure?

(a) Out of the page (b) Into the page (c) Up the page (d) Down the page
15. When a 4V battery is connected across an unknown resistor there is a current of 100 mA in the
circuit. The value of the resistance of the resistor is:
(a) 4 Ω (b) 40 Ω (c) 400 Ω (d) 0.4 Ω

16. A uniform magnetic field exists in the plane of paper pointing from left to right as shown in
below Figure. In the field an electron and a proton move as shown. The electron and the proton
experience
Prepared by: M. S. KumarSwamy, TGT(Maths) Page - 2 -
(a) forces both pointing into the plane of paper
(b) forces both pointing out of the plane of paper
(c) forces pointing into the plane of paper and out of the plane of paper, respectively
(d) force pointing opposite and along the direction of the uniform magnetic field respectively

DIRECTION: In the question number 17 and 20, a statement of Assertion (A) is followed by a
statement of Reason (R).
Choose the correct option
(a) Both assertion (A) and reason (R) are true and reason (R) is the correct explanation of
assertion (A)
(b) Both assertion (A) and reason (R) are true and reason (R) is not the correct explanation of
assertion (A)
(c) Assertion (A) is true but reason (R) is false.
(d) Assertion (A) is false but reason (R) is true.
17. Assertion (A): A compass needle is placed near a current carrying wire. The deflection of the
compass needle decreases when the magnitude of an electric current in the wire is increased.
Reason (R): Strength of a magnetic field at a point near the conductor increases on increasing
the current.
18. Assertion (A): Hydrogen gas is not evolved when a metal reacts with nitric acid.
Reason (R): Nitric acid is a strong oxidising agent.
19. Assertion(A): The sex of a child in human beings will be determined by the type of
chromosome he/she inherits from the father.
Reason(R): A child who inherits ‘X’ chromosome from his father would be a girl (XX), while a
child who inherits a ‘Y’ chromosome from the father would be a boy (XY).
20. Assertion(A): Spores are formed in sporangia.
Reason(R): Spores grow into separate individuals in moist conditions.

SECTION – B
Questions 21 to 25 carry 2 marks each.
21. Identify the substances oxidised and the substances reduced in the following reactions.
(i) ZnO(s) + C(s) → Zn + CO(g)
(ii) 4Na(s) + O2(g) → 2Na2O(s)
OR
Which among the following changes are exothermic or endothermic in nature?
(i) Decomposition of ferrous sulphate (ii) Dilution of sulphuric acid
(iii) Dissolution of sodium hydroxide in water (iv) Dissolution of ammonium chloride in water
22. List in tabular form three distinguishing features between cerebrum and cerebellum.
23. What will happen if mucus is not secreted by the gastric glands?
24. Answer the following:
(i) Which hormone is responsible for the changes noticed in females at puberty?
(ii) Dwarfism results due to deficiency of which hormone?
(iii) Blood sugar level rises due to deficiency of which hormone?
(iv) Iodine is necessary for the synthesis of which hormone?

Prepared by: M. S. KumarSwamy, TGT(Maths) Page - 3 -


25. Give two examples of decomposers. State their important role in nature.
26. A student sitting at the back of the classroom cannot read clearly the letters written on the
blackboard. What advice will a doctor give to her? Draw ray diagram for the correction of this
defect.
OR
Why do we see a rainbow in the sky only after rainfall?

SECTION – C
Questions 27 to 33 carry 3 marks each.
27. Identify the acid and the base from which sodium chloride is obtained. Which type of salt is it?
When is it called rock salt? How is rock salt formed?
28. An element A burns with golden flame in air. It reacts with another element B, atomic number
17 to give a product C. An aqueous solution of product C on electrolysis gives a compound D
and liberates hydrogen. Identify A, B, C and D. Also write down the equations for the reactions
involved.
29. A lens produces a magnification of –0.5. Is this a converging or diverging lens? If the focal
length of the lens is 6 cm, draw a ray diagram showing the image formation in this case.
30. An object 6 cm in size is placed at 50 cm in front of a convex lens of focal length 30 cm. At
what distance from the lens should a screen be placed in order to obtain a sharp image of the
object? Find the nature and size of the image. Also draw labelled ray diagram to show the image
formation in this case.
31. Bile juice does not have any digestive enzyme but still plays a significant role in the process of
digestion. Justify the statement.
OR
What are the adaptations of leaf for photosynthesis?
32. (i) What is an electromagnet? List any two uses.
(ii) Draw a labelled diagram to show how an electromagnet is made.
OR
With the help of a labelled diagram, explain the distribution of magnetic field due to a current
through a circular loop. Why is it that if a current carrying coil has n turns the field produced at
any point is n times as large as that produced by a single turn?
33. You have been selected to talk on “ozone layer and its protection” in the school assembly on
‘Environment Day’.
(a) Why should ozone layer be protected to save the environment?
(b) List any two ways that you would stress in your talk to bring in awareness amongst your
fellow friends that would also help in protection of ozone layer as well as the environment.

SECTION – D
Questions 34 to 36 carry 5 marks each.

34. A metal carbonate X on reacting with an acid gives a gas which when passed through a solution
Y gives the carbonate back. On the other hand, a gas G that is obtained at anode during
electrolysis of brine is passed on dry Y, it gives a compound Z, used for disinfecting drinking
water. Identify X, Y, G and Z.
OR
An organic compound A on heating with concentrated H2SO4 forms a compound B which on
addition of one mole of hydrogen in presence of Ni forms a compound C. One mole of
compound C on combustion forms two moles of CO2 and 3 moles of H2O. Identify the
compounds A, B and C and write the chemical equations of the reactions involved.
35. (i) How will you infer with the help of an experiment that the same current flows through every
part of the circuit containing three resistors R1, R2 and R3 in series connected to a battery of V
volts?
Prepared by: M. S. KumarSwamy, TGT(Maths) Page - 4 -
(ii) Study the following circuit and find out:

(a) Current in 12  resistor. (b) Difference in the readings of A1 and A2, if any.
36. Explain Mendel’s experiment with peas on inheritance of characters considering only one
visible contrasting character.
OR
In the following crosses write the characteristics of the progeny.
Cross Progeny
(i) RR YY × RR YY ___________________________
Round, yellow and round, yellow
(ii) Rr Yy × Rr Yy ___________________________
Round, yellow and round, yellow
(iii) rr yy × rr yy ___________________________
Wrinkled, green and wrinkled, green
(iv) RR YY × rr yy ___________________________
Round, yellow and wrinkled, green

SECTION – E(Case Study Based Questions)


Questions 37 to 39 carry 4 marks each.
37. Case Study – 1
The ability of carbon atoms to form chains leads to the existence of a series of compounds that
have same functional group (and hence similar chemical properties) and only differ from each
other by the presence of an additional carbon atom and its two associated hydrogen atoms in the
molecule (which causes the physical properties to change in a regular manner). A series of
compounds related in this way is said to form an homologous series.

The point about chemical properties is best illustrated by the sections that follow, on different
homologous series. The changes in physical properties are a result of the changes that occur in
the strength of van der Waals’ forces with increasing molar mass and in some cases a change in
molecular polarity. The simplest illustration of the effect of chain length on physical properties
is the, variation of the boiling point of the alkanes with the number of carbon atoms in the chain,
as illustrated in figure.
Prepared by: M. S. KumarSwamy, TGT(Maths) Page - 5 -
(i) All the members of a homologous series have similar chemical properties. Why?
(ii) In the graph shown, which has the higher boiling point and why?
Hydrocarbon with 3 carbon atoms or hydrocarbon with 6 carbon atoms
(iii) (a) What is the boiling point of heptane as shown in the graph?
(b) Why the curve is initially steep and flattens at the end?
OR
(iii) (a) Write the molecular formula of the 2nd and the 3rd member of the homologous series
whose first member is methane.
(b) Write the next homologue of each of the following: I. C2H4 II. C4H6

38. Case Study – 2


Sahil performed an experiment to study the inheritance pattern of genes. He crossed tall pea
plants (TT) with short pea plants (tt) and obtained all tall plants in F1 generation.
(i) What will be set of genes present in the F1 generation?
(ii) Give reason why only tall plants are observed in F1 progeny.
(iii) When F1 plants were self-pollinated, a total of 800 plants were produced. How many of
these would be tall, medium height or short plants? Give the genotype of F2 generation.
OR
(iii) When F1 plants were cross - pollinated with plants having tt genes, a total of 800 plants
were produced. How many of these would be tall, medium height or short plants? Give the
genotype of F2 generation.

39. Case Study – 3


Light spectrum is the many different wavelengths of energy produced by light source. Light is
measured in nanometers (nm). Each nanometer represents a wavelength of light or band of light
energy. Visible light is the part of spectrum from 380 nm to 780 nm.

Isaac Newton was the first to use a glass prism to obtain the spectrum of sunlight. He tried to
split the colours of the spectrum of white light further by using another similar prism. He then
placed a second identical prism in an inverted position with respect to the first prism. This
allowed all the colours of the spectrum to pass through second prism. He found a beam of white
light emerging from the other side of the second prism. This observation gave Newton the idea
that the sunlight is made up of seven colours.
(i) (a) What is the range of wavelength of visible light spectrum?
(b) What do you understand by light spectrum?
(ii) Explain the process of refraction when critical angle between an equilateral prism and air is
45 degree, if the incident ray is perpendicular to the refracting surface?
OR
(ii) (a) Why do different rays deviate differently in the prism?
(b) How will you use two identical prisms so that a narrow beam of white light incident on one
prism emerges out of the second prism as white light?

Prepared by: M. S. KumarSwamy, TGT(Maths) Page - 6 -


KENDRIYA VIDYALAYA GACHIBOWLI, GPRA CAMPUS, HYD-32
SAMPLE PAPER TEST 06 FOR BOARD EXAM 2023

SUBJECT: SCIENCE MAX. MARKS: 80


CLASS : X DURATION: 3 HRS
General Instruction:
1. This Question Paper has 5 Sections A-E.
2. Section A has 20 MCQs carrying 1 mark each.
3. Section B has 5 questions carrying 02 marks each.
4. Section C has 6 questions carrying 03 marks each.
5. Section D has 4 questions carrying 05 marks each.
6. Section E has 3 case based integrated units of assessment (04 marks each) with sub-parts of the
values of 1, 1 and 2 marks each respectively.
7. All Questions are compulsory. However, an internal choice in 2 Qs of 5 marks, 2 Qs of 3 marks
and 2 Questions of 2 marks has been provided. An internal choice has been provided in the
2marks questions of Section E
8. Draw neat figures wherever required. Take π =22/7 wherever required if not stated.
SECTION – A
Questions 1 to 20 carry 1 mark each.
1. Beams of light are incident through the holes A and B and emerge out of box through the holes
C and D respectively as shown in figure.

Which of the following could be inside the box?


(a) A rectangular glass slab (b) A convex lens (c) A concave lens (d) A prism

2. During the preparation of hydrogen chloride gas on a humid day, the gas is usually passed
through the guard tube containing calcium chloride. The role of calcium chloride taken in the
guard tube is to:
(a) absorb the evolved gas (b) moisten the gas
(c) absorb moisture from the gas (d) absorb Cl– ions from the evolved gas

3. Generally, metals react with acids to give salt and hydrogen gas. Which of the following acids
does not give hydrogen gas on reacting with metals (except Mn and Mg)?
(a) H2 SO4 (b) HCl (c) HNO3 (d) All of these
Alkaline KMnO 4
4. CH3 — CH2 — OH  Heat
CH3 — COOH
In the above given reaction, alkaline KMnO4 acts as
(a) reducing agent (b) oxidising agent (c) catalyst (d) dehydrating agent

5. Sodium hydrogencarbonate when added to acetic acid evolves a gas.


Which of the following statements are true about the gas evolved?
(i) It turns lime water milky
(ii) It extinguishes a burning splinter
Prepared by: M. S. KumarSwamy, TGT(Maths) Page - 1 -
(iii) It dissolves in a solution of sodium hydroxide
(iv) It has a pungent odour
(a) (i) and (ii) (b) (i), (ii) and (iii) (c) (ii), (iii) and (iv) (d) (i) and (iv)

6. In the reaction of iron with copper sulphate solution:


CuSO4 + Fe → Cu + FeSO4
Which option in the given table correctly represents the substance oxidised and the reducing
agent?
OPTION Substance Oxidized Reducing Agent
(a) Fe Fe
(b) Fe FeSO4
(c) Cu Fe
(d) CuSO4 Fe

7. Given below is a diagrammatic representation of a process taking place in the human body.

In which of these regions/organs could it be occurring?


(i) lungs (ii) heart (iii) brain
(a) only in (i) (b) only in (ii) (c) only in (i) and (ii) (d) in all - (i), (ii) and (iii)

8. Choose the correct path of urine in our body


(a) kidney → ureter → urethra → urinary bladder
(b) kidney → urinary bladder → urethra → ureter
(c) kidney → ureters → urinary bladder → urethra
(d) urinary bladder → kidney → ureter → urethra

9. In peas, a pure tall plant (TT) is crossed with a short plant (tt). The ratio of pure tall plants to
short plants in F2 is
(a) 1 : 3 (b) 3 : 1 (c) 1 : 1 (d) 2 : 1

10. Which of the following statements is true?


(a) A convex lens has 4 dioptre power having a focal length 0.25 m
(b) A convex lens has –4 dioptre power having a focal length 0.25 m
(c) A concave lens has 4 dioptre power having a focal length 0.25 m
(d) A concave lens has –4 dioptre power having a focal length 0.25 m

Prepared by: M. S. KumarSwamy, TGT(Maths) Page - 2 -


11. The image shows the ray diagram of a defected eye.

Which option shows the correction of the defect of the eye?

(a)

(b)

(c)

(d) None of these

12. A conducting wire carries 1021 electrons in 4 minutes. What is the current flowing through the
wire?
(a) 40 A (b) 7 A (c) 4 A (d) 0.7 A

13. An electric toaster has a power rating of 200 W. It operates for 1 hour in the morning and 1 hour
in the evening. How much does it cost to operate the toaster for 10 days at Rs. 5 per kW h?
(a) Rs. 20 (b) Rs. 400 (c) Rs. 5000 (d) Rs. 10000

14. For a current in a long straight solenoid N and S poles are created at the two ends. Among the
following statement, the incorrect statement is
(a) The field lines inside the solenoids are in the form of straight lines which indicate that the
magnetic field is same at all points in the solenoid.
(b) The strong magnetic field produced inside the solenoid can be used to magnetise a piece of
magnetic material, when soft iron is placed inside the coil.
(c) The pattern of magnetic field associated with the solenoid is different from the pattern of the
magnetic field around a bar magnet.
(d) The N and S poles exchange positions when the direction of current through the solenoid is
reversed.
15. In the given food chain, suppose the amount of energy at the fourth trophic level is 5 kJ, what
will be the energy available at the producer level?
Grass → Grasshopper → Frog → Snake → Hawk
(a) 5 k J (b) 50 k J (c) 500 k J (d) 5000 k J
16. The percentage of solar radiation absorbed by all the green plants for the process of
photosynthesis is about
(a) 1 % (b) 5 % (c) 8 % (d) 10 %

Prepared by: M. S. KumarSwamy, TGT(Maths) Page - 3 -


DIRECTION: In the question number 17 and 20, a statement of Assertion (A) is followed by a
statement of Reason (R).
Choose the correct option
(a) Both assertion (A) and reason (R) are true and reason (R) is the correct explanation of
assertion (A)
(b) Both assertion (A) and reason (R) are true and reason (R) is not the correct explanation of
assertion (A)
(c) Assertion (A) is true but reason (R) is false.
(d) Assertion (A) is false but reason (R) is true.

17. Assertion (A): Myopia is the defect of the eye in which only nearer objects are seen by the eye.
Reason (R): The eye ball is elongated.

18. Assertion (A): In electrolysis of water, the volume of hydrogen liberated is twice the volume of
oxygen formed.
Reason (R): Water (H2O) has hydrogen and oxygen in the ratio of 1 : 2 by volume.

19. Assertion(A): If one bacterium divides then two resultant bacteria divide again to generate four
individual bacteria which will be highly identical.
Reason(R): It is due to asexual reproduction.

20. Assertion (A): The mirrors used in search lights are concave spherical.
Reason (R): In concave spherical mirror the image formed is always virtual.

SECTION – B
Questions 21 to 25 carry 2 marks each.

21. Which among the following are physical or chemical changes?


(i) Evaporation of petrol
(ii) Burning of Liquefied Petroleum Gas (LPG)
(iii) Heating of an iron rod to red hot
(iv) Curdling of milk
(v) Sublimation of solid ammonium chloride

22. An electric oven of 2 kW power rating is operated in a domestic electrical circuit of 220 V that
has a current rating of 5 A. What result do you expect? Explain.
OR
Two lamps, one rated 100 W; 220 V, and the other 60 W; 220 V, are connected in parallel to
electric mains supply. Find the current drawn by two bulbs from the line, if the supply voltage is
220 V.

23. What is the role of saliva in the digestion of food?

24. What is translocation? Why is it essential for plants?

25. Give the pair of contrasting traits of the following characters in pea plant and mention which is
dominant and recessive.
(i) yellow seed (ii) round seed

26. (i) Write the number of covalent bonds in the molecule of propane, C3H8.
(ii) Which element exhibits the property of catenation to maximum extent and why?
OR
Catenation is the ability of an atom to form bonds with other atoms of the same element. It is
exhibited by both carbon and silicon. Compare the ability of catenation of the two elements.
Give reasons.
SECTION – C
Prepared by: M. S. KumarSwamy, TGT(Maths) Page - 4 -
Questions 27 to 33 carry 3 marks each.
27. A silver article generally turns black when kept in the open for a few days. The article when
rubbed with toothpaste again starts shining.
(i) Why do silver articles turn black when kept in the open for a few days? Name the
phenomenon involved.
(ii) Name the black substance formed and give its chemical formula.

28. Size of image of an object formed by a mirror having a focal length of 20 cm, is observed to be
reduced to 1/3rd of its size. At what distance the object has been placed from the mirror? What
is the nature of the image and the mirror?

29. In an industrial process used for the manufacture of sodium hydroxide, a gas ‘A’ is formed as a
by-product. The gas ‘A’ reacts with lime water to give a compound ‘B’ which is used as a
bleaching agent in the chemical industry. Identify ‘A’ and ‘B’. Also give the chemical equations
of the reactions involved.
OR
A metal A, which is used in thermite process, when heated with oxygen gives an oxide B, which
is amphoteric in nature. Identify A and B. Write down the reactions of oxide B with HCl and
NaOH.
30. What are reflex actions? Give two examples. Explain a reflex arc.

31. Read the following information:


I. Resistivity of copper is lower than that of aluminium which in turn is lower than that of
constantan.
II. Six wires labelled as A, B, C, D, E, F have been designed as per the following parameters:
Wire Length Diameter Material Resistance
A l 2d Aluminium R1
B 2l d/2 Constantan R2
C 3l d/2 Constantan R3
D l/2 3d Copper R4
E 2l 2d Aluminium R5
F l/2 4d Copper R6
Answer the following questions using the above data:
(i) Which of the wires has maximum resistance and why?
(ii) Which of the wires has minimum resistance and why?
(iii) Arrange R1, R3 and R5 in ascending order of their values. Justify your answer.
32. The flow of energy between various components of the environment has been extensively
studied. Give an outline of the findings.
33. (i) Why are budding, fragmentation and regeneration all considered as asexual types of
reproduction?
(ii) With neat diagrams explain the process of regeneration in Planaria.
OR
Reproduction is one of the most important characteristics of living beings. Give three reasons in
support of the statement.

SECTION – D
Questions 34 to 36 carry 5 marks each.

34. Why are certain compounds called hydrocarbons? Write the general formula for homologous
series of alkanes, alkenes and alkynes and also draw the structure of the first member of each
series. Write the name of the reaction that converts alkenes into alkanes and also write a
chemical equation to show the necessary conditions for the reaction to occur.

Prepared by: M. S. KumarSwamy, TGT(Maths) Page - 5 -


35. Define pollination. Explain the different types of pollination. List two agents of pollination.
How does suitable pollination lead to fertilization?
OR
(i) Write the reaction that occurs when glucose breaks down anaerobically in yeast.
(ii) Write the mechanism by which fishes breathe in water.
(iii) Name the balloon like structures present in lungs. List its two functions.
(iv) Name the respiratory pigment and write its role in human beings.
36. Draw the pattern of the field lines of the magnetic field around a current carrying straight
conductor passing through and held perpendicular to a horizontal cardboard. State right-hand
thumb rule and explain how this rule is useful to determine the direction of the magnetic field in
the above case, if the direction of current in the conductor is vertically downwards.
OR
With the help of a labelled diagram, explain the distribution of magnetic field due to a current
through a circular loop. Why is it that if a current carrying coil has n turns the field produced at
any point is n times as large as that produced by a single turn?

SECTION – E(Case Study Based Questions)


Questions 37 to 39 carry 4 marks each.
37. Case Study – 1
For an internal combustion engine to move a vehicle down the road, it must convert the energy
stored in the fuel into mechanical energy to drive the wheels. In your car, the distributor and
battery provide this starting energy by creating an electrical “spark”, which helps in combustion
of fuels like gasoline. Below is the reaction depicting complete combustion of gasoline in full
supply of air: 2C8H18 (l) + 25 O2 (g) → 16 X + 18 Y
(i) Identify the types of chemical reaction occurring during the combustion of fuels? Name the
product ‘X’ and ‘Y’.
(ii) ‘Although nitrogen is the most abundant gas in the atmosphere, it does not take part in
combustion’. Justify the statement.
(iii) ‘A student while walking on the road observed that a cloud of black smoke belched out
from the exhaust stack of moving trucks on the road.’ Give reason.
OR
(iii) Write the balanced chemical equations for the following reaction and identify the type of
reaction: Ethene is burnt in the presence of oxygen to form carbon dioxide, water and releases
heat and light.
38. Case Study – 2
Humans use sexual mode of reproduction. But the actual transfer of germ cells between two
people needs special organs for the sexual act. In mammals such as humans, the baby is carried
in the mother’s body for a long period and is breastfed later on. The female reproductive organs
and breasts will need to mature to accommodate these possibilities. Hence some specialised
systems are involved in the process of sexual reproduction. The given graph shows the
hormonal changes during a normal menstrual cycle.

Prepared by: M. S. KumarSwamy, TGT(Maths) Page - 6 -


(i) What would be a likely consequence if the hormone represented by graph Q is lacking in an
adult female?
(ii) What is funeral of unfertilised egg?
(iii) After the beginning of menstrual cycle, at which day progesterone reaches its peak? Give
reason.
OR
(iii) (a) Name two simple organisms having the ability of regeneration.
(b) What is the role of the seminal vesicles and the prostate gland?

39. Case Study - 3


The image formed by a convex lens depends on the position of the object in front of the lens.
When the object is placed anywhere between focus and infinity, the image formed by convex
lens is real and inverted. The image is not obtained on the screen when the object is placed
between focus and the lens.
The distance between the optical centre O of the convex lens and the focus point F1 or F2 is its
focal length.

When the object shifts from –∞ to F1, the image moves from F2 to +∞.

When the object shifts from F1 to O, the image moves from –∞ to O.

A student did an experiment with a convex lens. He put an object at different distances from the
lens. In each case he measured the distance of the image from the lens. The results were
recorded in the following table.
Object distance (in cm) 25 30 40 60 120
Image distance (in cm) 100 24 60 30 40
Unfortunately, his results are written in the wrong order.
(i) Arrange the image distance in the correct order (in cm).
(ii) Which of the object distances gives the biggest image? Give reason.
(iii) Find the focal length of this lens.
OR
(iii) What is the minimum distance between an object and its real image formed by a convex
lens? Where should an object be placed to get a virtual image by convex lens?

Prepared by: M. S. KumarSwamy, TGT(Maths) Page - 7 -


KENDRIYA VIDYALAYA GACHIBOWLI, GPRA CAMPUS, HYD-32
SAMPLE PAPER TEST 07 FOR BOARD EXAM 2023

SUBJECT: SCIENCE MAX. MARKS: 80


CLASS : X DURATION: 3 HRS
General Instruction:
1. This Question Paper has 5 Sections A-E.
2. Section A has 20 MCQs carrying 1 mark each.
3. Section B has 5 questions carrying 02 marks each.
4. Section C has 6 questions carrying 03 marks each.
5. Section D has 4 questions carrying 05 marks each.
6. Section E has 3 case based integrated units of assessment (04 marks each) with sub-parts of the
values of 1, 1 and 2 marks each respectively.
7. All Questions are compulsory. However, an internal choice in 2 Qs of 5 marks, 2 Qs of 3 marks
and 2 Questions of 2 marks has been provided. An internal choice has been provided in the
2marks questions of Section E
8. Draw neat figures wherever required. Take π =22/7 wherever required if not stated.
SECTION – A
Questions 1 to 20 carry 1 mark each.
1. Pentane has the molecular formula C5 H12. It has
(a) 5 covalent bonds (b) 12 covalent bonds
(c) 16 covalent bonds (d) 17 covalent bonds

2. A scientist is attempting to represent an ionic bond between calcium and chlorine. The figure
below shows the progress he has made so far.

What should be the next step in his representation of the ionic bond?
(a) Transfer an electron from the calcium atom to the chlorine atom.
(b) Transfer an electron from the chlorine atom to the calcium atom.
(c) Add another chlorine atom to accept an electron from the calcium atom.
(d) Add another calcium atom to donate an electron to the chlorine atom.
3. The composition of aqua regia is:
(a) Dil. HCl : Conc. HNO3 = 3 : 1 (b) Conc. HCl : Dil. HNO3 = 3 : 1
(c) Conc. HCl : Conc. HNO3 = 3 : 1 (d) Dil. HCl : Dil. HNO3 = 3 : 1
4. Which among the following statement(s) is (are) true?
Exposure of silver chloride to sunlight for a long duration turns grey due to:
(i) The formation of silver by decomposition of silver chloride.
(ii) Sublimation of silver chloride.
(iii) Decomposition of chlorine gas from silver chloride.
(iv) Oxidation of silver chloride.
(a) (i) only (b) (i) and (iii) (c) (ii) and (iii) (d) (iv) only

Prepared by: M. S. KumarSwamy, TGT(Maths) Page - 1 -


5. A sample of soil is mixed with water and allowed to settle. The clear supernatant solution turns
the pH paper yellowish-orange. Which of the following would change the colour of this pH
paper to greenish-blue?
(a) Lemon juice (b) Vinegar (c) Common salt (d) An antacid

6. Which of the following reactions is a neutralisation reaction?


(a) 4Na + Cl2 → 2Na2O (b) Fe + 2HCl → FeCl2 + H2
(c) MgO + H2O → Mg(OH)2 (d) HNO3 + NaOH → NaNO2 + H2O

7. Identify the unsaturated compounds from the following


(i) Propane (ii) Propene (iii) Propyne (iv) Chloropropane
(a) (i) and (ii) (b) (ii) and (iv) (c) (iii) and (iv) (d) (ii) and (iii)

8. The figure given below shows a schematic plan of blood circulation in humans with labels (i) to
(iv). Identify the correct label with its functions?

(a) (i) Pulmonary vein - takes impure blood from body part.
(b) (ii) Pulmonary artery - takes blood from lung to heart.
(c) (iii) Aorta - takes blood from heart to body parts.
(d) (iv) Vena cava - takes deoxygenated blood from body parts to right auricle.
9. Which among the following statements are true for sexual reproduction in flowering plants?
(i) It requires two types of gametes.
(ii) Fertilisation is a compulsory event.
(iii) It always results in formation of zygote.
(iv) Offspring formed are clones.
(a) (i) and (iv) (b) (i), (iii) and (iv)
(c) (i), (ii) and (iii) (d) (i), (ii) and (iv)
10. In a neuron, conversion of electrical signal to a chemical signal occurs at/in:
(a) cell body (b) axonal end (c) dendritic end (d) axon
11. Length of pollen tube depends on the distance between:
(a) pollen grain and upper surface of stigma
(b) pollen grain on upper surface of stigma and ovule
(c) pollen grain in anther and upper surface of stigma
(d) upper surface of stigma and lower part of style
12. A zygote which has an X-chromosome inherited from the father will develop into a
(a) Boy (b) Girl
(c) X-chromosome does not determine the sex of a child (d) either boy or girl

Prepared by: M. S. KumarSwamy, TGT(Maths) Page - 2 -


13. In an electrical circuit, three incandescent bulbs A, B, and C of rating 40 W, 100 W, and 60 W,
respectively are connected in parallel to an electric source. Which of the following is likely to
happen regarding their brightness?
(a) Brightness of all the bulbs will be the same.
(b) Brightness of bulb A will be the maximum.
(c) Brightness of bulb B will be more than that of A and C.
(d) Brightness of bulb C will be less than that of B.

14. A student learns that magnetic field strength around a bar magnet is different at every point.
Which diagram shows the correct magnetic field lines around a bar magnet?
(a) (b)

(c) (d)

15. If the field lines in a magnetic field are parallel and equidistant, the magnetic field is
(a) Uniform (b) Non uniform (c) Zero (d) None of these

16. The angle of incidence from air to glass at the point O on the hemispherical glass slab is

(a) 45° (b) 0° (c) 90° (d) 180°

DIRECTION: In the question number 17 and 20, a statement of Assertion (A) is followed by a
statement of Reason (R).
Choose the correct option
(a) Both assertion (A) and reason (R) are true and reason (R) is the correct explanation of
assertion (A)
(b) Both assertion (A) and reason (R) are true and reason (R) is not the correct explanation of
assertion (A)
(c) Assertion (A) is true but reason (R) is false.
(d) Assertion (A) is false but reason (R) is true.
Ans: (b) Both assertion (A) and reason (R) are true and reason (R) is not the correct explanation
of assertion (A)

Prepared by: M. S. KumarSwamy, TGT(Maths) Page - 3 -


17. Assertion (A) : Cyton region of nerve fibre collects information for the brain.
Reason (R) : Nerve fibres can either have or lack myelin sheath.

18. Assertion (A): The wires supplying current to an electric heater are not heated appreciably.
Reason (R): Resistance of connecting wires is very small and H<< R.

19. Assertion (A): Electrolysis of concentrated solution of sodium chloride is known as chlor-alkali
solution.
Reason (R): The products formed are hydrogen, chlorine and sodium hydroxide.

20. Assertion (A): At puberty, in boys, voice begins to crack and thick hair grows on face.
Reason (R): At puberty, there is decreased secretion of testosterone in boys.

SECTION – B
Questions 21 to 26 carry 2 marks each.

21. The electric power consumed by a device may be calculated by the using either of the two
V2
expressions P = I2R or P  . The first expression indicates that it is directly proportional to
R
R whereas the second expression indicates inverse proportionality. How can the seemingly
different dependence of P on R in these expressions be explained.
OR
Consider the scale of a voltmeter shown in the diagram and answer the following questions:

(i) What is the least count of the voltmeter?


(ii) What is the reading shown by the voltmeter?
(iii) If this voltmeter is connected across a resistor of 20 W, how much current is flowing
through the resistor?
22. What is the role of metal or reagents written on arrow’s in the given chemical reactions?

(a)

(b)
OR
The molecular formula of an organic compound X is C2H4O2 which has vinegar - like smell.
(a) Identify the compound.
(b) Write its chemical formula and name.
23. Draw a well labelled diagram of stomata. List two functions of stomata.

Prepared by: M. S. KumarSwamy, TGT(Maths) Page - 4 -


24. Why is KOH solution kept in the test-tube inside the airtight conical flask while doing the
experiment of respiration of seeds?
25. While teaching the chapter "Our Environment", the teacher stressed upon the harmful effects of
burning of fossil fuels, plastic paper, etc. The students noticed the extensive use of plastic and
polythene in daily life, which can be avoided and the surroundings can be kept clean. They
decided to make their school "Plastic and Polythene" free and motivated each other for its
minimum use.
(a) Why should the use of polythene and plastic be reduced in daily life?
(b) In what way the students would have avoided the use of plastic and polythene in their
school?
26. List two differences in tabular form between dominant trait and recessive trait. What
percentage/proportion of the plants in the F2 generation/progeny were round, in Mendel’s cross
between round and wrinkled pea plants?

SECTION – C
Questions 27 to 33 carry 3 marks each.
27. What are covalent compounds? How are they different from ionic compounds? List any two
properties of covalent compounds.
28. Name the hormone which regulates carbohydrate, protein and fat metabolism in our body.
Which gland secretes this hormone? Why is it important for us to have iodised salt in our diet?
OR
Reproduction is one of the most important characteristics of living beings. Give three reasons in
support of the statement.
29. If the image formed by a lens for all positions of an object placed in front of it is always erect
and diminished, what is the nature of this lens? Draw a ray diagram to justify your answer. If the
numerical value of the power of this lens is 10D, what is its focal length in the Cartesian
system?
30. (a) For the preparation of cakes, baking powder is used. If at home, your mother uses baking
soda instead of baking powder, how will it affect the taste of the cake and why?
(b) How is baking soda be converted into baking powder?
(c) What makes the cake soft and spongy?
OR
(i) Which of the following reactions is/are an endothermic reaction(s) where decomposition also
happens?
(a) Respiration (b) Heating of lead nitrate
(c) Decomposition of organic matter (d) Electrolysis of acidified water
(ii) Silver chloride when kept in the open turns grey. Illustrate this with a balanced chemical
equation.
31. Refractive index of water with respect to air is 1.33 and that of diamond is 2.42.
(a) In which medium does the light move faster, water or diamond?
(b) What is the refractive index of diamond with respect to water?
32. (a) From the following groups of organisms, create a food chain which is the most advantageous
for Human being in terms of energy.

Prepared by: M. S. KumarSwamy, TGT(Maths) Page - 5 -


(b) State the possible disadvantage if the cereal plant is growing in a soil rich in pesticides.
(c) Construct a food web using the organisms mentioned above.

33. The figure below shows three cylindrical copper conductors along with their face areas and
lengths. Compare the resistance and the resistivity of the three conductors. Justify your answer.

SECTION – D
Questions 34 to 36 carry 5 marks each.

34. Explain the process of nutrition in Amoeba.


OR
Give reasons:
(a) Ventricles have thicker muscular walls than atria.
(b) Transport system in plants is slow.
(c) Circulation of blood in aquatic vertebrates differs from that in terrestrial vertebrates.
(d) During the daytime, water and minerals travel faster through xylem as compared to the
night.
(e) Veins have valves whereas arteries do not.

35. It is desired to obtain an erect image of an object, using concave mirror of focal length of 12 cm.
(i) What should be the range of distance of an object placed in front of the mirror?
(ii) Will the image be smaller or larger than the object. Draw ray diagram to show the formation
of image in this case.
(iii) Where will the image of this object be, if it is placed 24 cm in front of the mirror?
Draw ray diagram for this situation also to justify your answer.
Show the positions of pole, principal focus and the centre of curvature in the above ray
diagrams.
OR
(i) Amit claims to have obtained an image twice the size of object with a concave lens. Is he
correct? Give reason for your answer.
(ii) Where should an object be placed in case of a convex lens to form an image of same size as
of the object? Show with the help of ray diagram the position and the nature of the image
formed.
(iii) With the help of ray diagram, illustrate the change in position, nature and size of the image
formed if the convex lens in case of (ii) is replaced by concave lens of same focal length.

36. (a) Define corrosion.


(b) What is corrosion of iron called?
(c) How will you recognise the corrosion of silver?
(d) Why corrosion of iron is a serious problem?
(e) How can we prevent corrosion of iron?

SECTION – E(Case Study Based Questions)


Questions 37 to 39 carry 4 marks each.
37. Case Study - 1
A student was asked to perform an experiment to study the force on a current carrying
conductor in a magnetic field. He took a small aluminium rod AB, a strong horse shoe magnet,
some connecting wires, a battery and a switch and connected them as shown. He observed that
Prepared by: M. S. KumarSwamy, TGT(Maths) Page - 6 -
on passing current, the rod gets displaced. On reversing the direction of current, the direction of
displacement also gets reversed.

On the basis of your understanding of this phenomenon, answer the following questions:
(i) Why does the rod get displaced on passing current through it?
(ii) State the rule that determines the direction of the force on conductor AB.
(iii) (a) In the above experimented set up, when current is passed through the rod, it gets
displaced towards the left. What will happen to the displacement if the polarity of the magnet
and the direction of current both are reversed?
(b) Name any two devices that use current carrying conductors and magnetic field.
OR
(iii) Draw the pattern of magnetic field lines produced around a current carrying straight
conductor held vertically on a horizontal cardboard. Indicate the direction of the field lines as
well as the direction of current flowing through the conductor.

38. Case Study – 2


The given diagram represents the human endocrine system. Study the diagram and answer any
four questions from Q. 1 to Q. 5.

(a) Identify the endocrine glands A, B, C, D, E and F in the given diagram.


(b) Which gland secretes digestive enzymes as well as hormones?
(c) What is the effect of chemical produced by gland F?
OR
(c) Name the endocrine glands which is unpaired? Explain

Prepared by: M. S. KumarSwamy, TGT(Maths) Page - 7 -


39. Case Study – 3
The pH of a solution is a measure of its hydrogen ion (H+) concentration. It is measured
generally using pH scale. The values on pH scale ranges from 0 to 14.
A pH of 1 is very acidic and corresponds to a high concentration of H+ ions. A pH of 14 is very
basic and corresponds to a low concentration of H+ ions. The pH of a neutral solution is 7. The
table given below shows the pH and H+ ion concentration of some common aqueous solutions.
The leftmost column shows the number of moles of H+ ions in 1 mole of liquid.
The pH and Hydrogen ion (H+) Concentration of Some Solutions

H+ Concentration
pH Solution
(moles)
10–1 1
10–2 2 Gastric (stomach) juice, cola, lemon juice
10–3 3 Vinegar
10–4 4 Tomato juice
10–5 5 Black coffee, rain water
10–6 6 Urine
10–7 7 Pure water
10–8 8 Sea water
10–9 9 Baking soda
10–10 10
10–11 11 Milk of magnesia
10–12 12 Household bleach
10–13 13 Oven cleaner
10–14 14

(i) How is the hydrogen ion concentration and pH related to each other?
(ii) On the basis of above table, arrange the following in the decreasing order of H+
concentration.
Pure water, tomato juice, milk of magnesia, sea water
(iii) A solution of pH 2 is filled in two separate beakers. A few drops of methyl orange and
phenolphthalein are added into separate solutions. How will the colour of the indicators change?
OR
(iii) Two solutions X and Y have pH values of 3.0 and 9.5 respectively. Which of these will turn
litmus solution from blue to red and which will turn phenolphthalein from colourless to pink?

Prepared by: M. S. KumarSwamy, TGT(Maths) Page - 8 -


KENDRIYA VIDYALAYA GACHIBOWLI, GPRA CAMPUS, HYD-32
SAMPLE PAPER TEST 08 FOR BOARD EXAM 2023

SUBJECT: SCIENCE MAX. MARKS: 80


CLASS : X DURATION: 3 HRS
General Instruction:
1. This Question Paper has 5 Sections A-E.
2. Section A has 20 MCQs carrying 1 mark each.
3. Section B has 5 questions carrying 02 marks each.
4. Section C has 6 questions carrying 03 marks each.
5. Section D has 4 questions carrying 05 marks each.
6. Section E has 3 case based integrated units of assessment (04 marks each) with sub-parts of the
values of 1, 1 and 2 marks each respectively.
7. All Questions are compulsory. However, an internal choice in 2 Qs of 5 marks, 2 Qs of 3 marks
and 2 Questions of 2 marks has been provided. An internal choice has been provided in the
2marks questions of Section E
8. Draw neat figures wherever required. Take π =22/7 wherever required if not stated.
SECTION – A
Questions 1 to 20 carry 1 mark each.
1. If a few drops of a concentrated acid accidentally spills over the hand of a student, what should
be done?
(a) Wash the hand with saline solution
(b) Wash the hand immediately with plenty of water and apply a paste of sodium hydrogen
carbonate
(c) After washing with plenty of water apply a solution of sodium hydroxide on the hand
(d) Neutralise the acid with a strong alkali

2. Which of the following represents saponification reaction?


CaO
(a) CH 3COONa  NaOH   CH 4  Na2CO3
H 2 SO4
(b) CH 3COOH  C2 H 5OH   CH 3COOC2 H 5  H 2O
(c) 2CH 3COOH  2 Na 
 2CH 3COONa  H 2
(d) CH 3COOC2 H 5  NaOH 
 CH 3COONa  C2 H 5OH

3. Which of the following statements is correct about an aqueous solution of an acid and of a base?
(i) Higher the pH, stronger the acid
(ii) Higher the pH, weaker the acid
(iii) Lower the pH, stronger the base
(iv) Lower the pH, weaker the base
(a) (i) and (iii) (b) (ii) and (iii) (c) (i) and (iv) (d) (ii) and (iv)

4. A student performs some activities on two substances and records the observations in a table as
shown.
Activity Substance M Substance N
Cut with a knife Forms small pieces Forms small pieces
Beaten with hammer Shape changes Changes into powder
Stricken with a metal rod Makes a sound Changes into powder
Which option classifies the substances into metals and non-metals?
(a) Both the substances are non-metals.
(b) Both the substances are metals.
(c) Substance M is metal while substance N is non-metal.
(d) Substance M is non-metal while substance N is metal.
Prepared by: M. S. KumarSwamy, TGT(Maths) Page - 1 -
5. What happens when calcium is treated with water?
(i) It does not react with water.
(ii) It reacts violently with water.
(iii) It reacts less violently with water.
(iv) Bubbles of hydrogen gas formed stick to the surface of calcium.
(a) (i) and (iv) (b) (ii) and (iii) (c) (i) and (ii) (d) (iii) and (iv)

6. Which of the following a balanced equation for the following chemical reaction:
Heat
(i) 2 KClO3   2 KCl  3O2 (ii) MgO  H 2O  Mg (OH ) 2
(iii) 4 Al  3O3 
 2 Al2O3 (iv) Zn  FeSO4  ZnSO4  Fe
(a) (i) and (iii) (b) (iii) and (iv) (c) (ii) and (iv) (d) (ii) and (iii)

7. Sodium carbonate is a basic salt because it is a salt of


(a) strong acid and strong base (b) weak acid and weak base
(c) strong acid and weak base (d) weak acid and strong base
8. Oxygen liberated during photosynthesis comes from
(a) water (b) chlorophyll (c) carbon dioxide (d) glucose
9. When we touch the leaves of “touch-me-not” plant, they begun to fold up and droop. How does
the plant communicate the information of touch?
(a) The plant uses electrical signals to transfer information from external environment to cells.
(b) The plant uses electrical- chemical signals to transfer information from cell to cell.
(c) The plant uses electrical- chemical signals to transfer information from tissue to specialized
cells.
(d) The plant uses electrical signals to transfer information from cell to specialized tissues.
10. Which one of the following device can detect the feeble current in the electrical circuit?
(a) Voltmeter (b) Ammeter (c) Galvanometer (d) Rheostat
11. Which of the following statement (s) is (are) true about nastic movements?
(i) These are slow movements.
(ii) These occur either towards or away from the stimulus.
(iii) These involve the use of electrochemical signals by the plants.
(iv) In such movements, the plant cells change shape by altering their water content.
(a) 1 and 2 only (b) 1 and 3 only (c) 3 and 4 only (d) 2 and 4 only
12. A current that always flows in the same direction through an electrical device is called
(a) a direct current. (b) an alternating current.
(c) a varying current. (d) a magnetic current.
13. Identify the option that indicates the correct enzyme that is secreted in location (i), (ii) and (iii).

:
Prepared by: M. S. KumarSwamy, TGT(Maths) Page - 2 -
(a) (i)—lipase, (ii)—trypsin, (iii)—pepsin
(b) (i)—amylase, (ii)—pepsin, (iii)—trypsin
(c) (i)—trypsin, (ii)—amylase, (iii)—carboxylase
(d) (i)—permease, (ii)—carboxylase, (iii)—oxidase
14. Ten identical wires each having a resistance of 1Ω are connected in parallel. The combination
will have a resistance of
(a) 10 Ω (b) 1 Ω (c) 0.1 Ω (d) 0.01 Ω
15. What is the direction of magnetic field at a point A above the wire carrying current I as shown
in figure?

(a) Out of the page (b) Into the page


(c) Up the page (d) Down the page
16. What is common between extensive network of blood vessels around walls of alveoli and in
glomerulus of nephron?
(a) Thick walled arteries richly supplied with blood
(b) Thin walled veins poorly supplied with blood
(c) Thick walled capillaries poorly supplied with blood
(d) Thin walled capillaries richly supplied with blood

DIRECTION: In the question number 17 and 20, a statement of Assertion (A) is followed by a
statement of Reason (R).
Choose the correct option
(a) Both assertion (A) and reason (R) are true and reason (R) is the correct explanation of
assertion (A)
(b) Both assertion (A) and reason (R) are true and reason (R) is not the correct explanation of
assertion (A)
(c) Assertion (A) is true but reason (R) is false.
(d) Assertion (A) is false but reason (R) is true.
17. Assertion (A) : Alloys are commonly used in electrical heating devices, like electrical iron,
toasters etc.
Reason (R) : Alloys do not oxidise (burn) readily at high temperatures.
18. Assertion (A) : Fresh milk in which baking soda is added, takes a longer time to set as curd.
Reason (R) : Baking soda decreases the pH value of fresh milk to below 6.
19. Assertion (A): Our body maintains blood sugar level.
Reason (R): Pancreas secretes insulin which helps to regulate blood sugar levels in the body.
20. Assertion (A) : Clones are offspring of an organism formed by asexual reproduction.
Reason (R) : Clones have exact copies of DNA as their parent.

SECTION – B
Questions 21 to 25 carry 2 marks each.
21. In the experiment ‘‘To prepare a temporary mount of a leaf peel to show stomata’’, glycerine
and safranin are used. When and why are these two liquids used? Explain.
OR
Trace the path of sperm during ejaculation and mention the gland and their functions associated
with the male reproductive system.

22. Sudha finds out that the sharp image of the window pane of her science laboratory is formed at a
distance of 15 cm from the lens. She now tries to focus the building visible to her outside the
window instead of the window pane without disturbing the lens. In which direction will she
Prepared by: M. S. KumarSwamy, TGT(Maths) Page - 3 -
move the screen to obtain a sharp image of the building? What is the approximate focal length
of this lens?
OR
Identify the device used as a spherical mirror or lens in the following cases, when the image
formed is virtual and erect in each case.
(a) Object is placed between device and its focus, image formed is enlarged and behind it.
(b) Object is placed between the focus and device, image formed is enlarged and on the same
side as that of the object.
(c) Object is placed between infinity and device, image formed is diminished and between focus
and optical centre on the same side as that of the object.
(d) Object is placed between infinity and device, image formed is diminished and between pole
and focus, behind it.

23. State the reason for the following:


(i) An iron strip dipped in a blue copper sulphate solution turns the blue solution pale green.
(ii) Calcium does not occur in free state in nature.

24. Distinguish between a gamete and zygote. Explain their roles in sexual reproduction.

25. Why is nutrition necessary for an organism? Give any two reasons.

26. Suggest one word for each of the following statements/ definitions
(a) The physical and biological world where we live in
(b) Each level of the food chain where the transfer of energy takes place
(c) The physical factors like temperature, rainfall, wind and soil of an ecosystem
(d) Organisms which depend on the producers either directly or indirectly for food

SECTION – C
Questions 27 to 33 carry 3 marks each.
27. What is feedback mechanism of hormonic regulation? Take the example of insulin to explain
this phenomenon.
OR
List in tabular form three distinguishing features between cerebrum and cerebellum.

28. (i) State the relation between colour of scattered light and size of the scattering particle.
(ii) The apparent position of an object, when seen through the hot air, fluctuates or wavers. State
the basic cause of this observation.
(iii) Complete the path of white light when it passes through two identical prisms placed as
shown:

29. Define the term power of accommodation. Write the modification in the curvature of the eye
lens which enables us to see the nearby objects clearly?

30. (i) With the help of a suitable circuit diagram prove that the reciprocal of the equivalent
resistance of a group of resistances joined in parallel is equal to the sum of the reciprocals of the
individual resistances.
Prepared by: M. S. KumarSwamy, TGT(Maths) Page - 4 -
(ii) In an electric circuit two resistors of 12 Ω each are joined in parallel to a 6 V battery. Find
the current drawn from the battery.

31. Write the essential function performed by ozone at the higher levels of the Earth’s atmosphere.
How is it produced? Name the synthetic chemicals mainly responsible for the drop of amount of
ozone in the atmosphere. How can the use of these chemicals be reduced?

32. Identify the type of each of the following reactions. Also write balanced chemical equation for
each reaction.
(i) A reaction in which the reaction mixture becomes warm.
(ii) A reaction in which an insoluble substance is formed.

33. 3 mL of ethanol is taken in a test tube and warmed gently in a water bath. A 5% solution of
alkaline potassium permanganate is added first drop by drop to this solution, then in excess.
(i) How is 5% solution of KMnO4 prepared?
(ii) State the role of alkaline potassium permanganate in this reaction. What happens on adding
it in excess?
(iii) Write chemical equation of this reaction.

SECTION – D
Questions 34 to 36 carry 5 marks each.
34. Draw the structure of a neuron and explain its function.
OR
Draw the diagram of a flower and label the four whorls. Write the names of gamete producing
organs in the flower.

35. (a) Draw a labelled circuit diagram to study the relationship between the current (I) flowing
through a conductor and the potential difference (V) applied across its two ends. State the
formula co-relating the I in a conductor and the V across it. Also, show their relationship by
drawing a diagram.
(b) What would be the resistance of a resistor if the current flowing through it is 0.15 A when
the potential difference across it is 1.05 V?
36. Give reasons:
(i) Reactivity of Al decreases if it is dipped in HNO3.
(ii) Carbon cannot reduce the oxides of Na or Mg.
(iii) NaCl is not a conductor of electricity in solid state whereas it does conduct electricity in
aqueous solution as well as in molten state.
(iv) Iron articles are galvanised.
(v) Metals like Na, K, Ca and Mg are never found in their free state in nature.
OR
(a) Write the chemical name of Na2CO3.10H2O and Na2CO3.
(b) Write the significance of 10H2O.
(c) Mention the term used for water molecules attached with a salt.
(d) With the help of a chemical equation, explain the method of preparation of both
Na2CO3.10H2O and Na2CO3. Also, list two uses of Na2CO3.10H2O.

SECTION – E(Case Study Based Questions)


Questions 37 to 39 carry 4 marks each.
37. Case Study - 1
Pihu passed a narrow beam PQ of white light through a glass prism ABC as shown in the
diagram.

Prepared by: M. S. KumarSwamy, TGT(Maths) Page - 5 -


(a) Trace it on your answer sheet and show the path of the emergent beam as observed on the
screen DE.
(b) Write the name and cause of the phenomenon observed.
(c) Where else in nature is this phenomenon observed?
OR
Based on this observation, state the conclusion which can be drawn about the constituents of
white light.
38. Case Study – 2
The given figure shows how the root and shoot of a seed develop in response to stimull.

(i) (a) Identify the stimulus that causes the roots to grow in the direction shown in figure.
(b) State the name of this response.
(ii) (a) Identify the stimulus that cause the shoot to grow in the direction shown in figure.
(b) State the name of this response.
OR
(ii) (a) State the name of the plant hormone that causes the root and shoot to develop in the way
shown in figure.
(b) What is hydrotropism?
39. Case Study – 3
A student focussed the image of a candle flame on a white screen by placing the flame at
various distances from a convex lens. He noted his observations as:
S. No. Distance of flame from Distance of Screen from
the lens (cm) the lens (cm)
(a) 60 20
(b) 40 24
(c) 30 30
(d) 24 40
(e) 15 70
(a) In which case the size of object and image will be same?
(b) What is the change in the image observed as the object is moved from infinity towards the
concave lens?
(c) From the above table, find the focal length of lens without using lens formula.
OR
Which set of observations is incorrect?

Prepared by: M. S. KumarSwamy, TGT(Maths) Page - 6 -


KENDRIYA VIDYALAYA GACHIBOWLI, GPRA CAMPUS, HYD-32
SAMPLE PAPER TEST 09 FOR BOARD EXAM 2023

SUBJECT: SCIENCE MAX. MARKS: 80


CLASS : X DURATION: 3 HRS
General Instruction:
1. This Question Paper has 5 Sections A-E.
2. Section A has 20 MCQs carrying 1 mark each.
3. Section B has 5 questions carrying 02 marks each.
4. Section C has 6 questions carrying 03 marks each.
5. Section D has 4 questions carrying 05 marks each.
6. Section E has 3 case based integrated units of assessment (04 marks each) with sub-parts of the
values of 1, 1 and 2 marks each respectively.
7. All Questions are compulsory. However, an internal choice in 2 Qs of 5 marks, 2 Qs of 3 marks
and 2 Questions of 2 marks has been provided. An internal choice has been provided in the
2marks questions of Section E
8. Draw neat figures wherever required. Take π =22/7 wherever required if not stated.
SECTION – A
Questions 1 to 20 carry 1 mark each.
1. Which of the following two combinations are correct?

Metal Gas evolved


(i) Copper Yes
(ii) Iron Yes
(iii) Magnesium No
(iv) Zinc Yes
(a) (i) and (iii) (b) (i) and (iv) (c) (ii) and (iii) (d) (ii) and (iv)

2. Which one of the following structures correctly depicts the compound CaCl2?

3. Identify the name of the given compound.


(a) Butanal (b) Butanone (c) Propanol (d) Propanal

4. Solid calcium oxide reacts vigorously with water to form calcium hydroxide accompanied by
liberation of heat. This process is called slaking of lime. Calcium hydroxide dissolves in water
to form its solution called lime water. Which among the following is (are) true about slaking of
lime and the solution formed?
(i) It is an endothermic reaction
Prepared by: M. S. KumarSwamy, TGT(Maths) Page - 1 -
(ii) It is an exothermic reaction
(iii) The pH of the resulting solution will be more than seven
(iv) The pH of the resulting solution will be less than seven
(a) (i) and (ii) (b) (ii) and (iii)
(c) (i) and (iv) (d) (iii) and (iv)

5. If a few drops of a concentrated acid accidentally spills over the hand of a student, what should
be done?
(a) Wash the hand with saline solution.
(b) Wash the hand immediately with plenty of water and apply a paste of sodium
hydrogencarbonate.
(c) After washing with plenty of water, apply solution of sodium hydroxide on the hand.
(d) Neutralise the acid with a strong alkali.

6. Identify the option that indicates the correct enzyme that is secreted in location i, ii and iii.

(a) (i)-lipase, (ii)-trypsin, (iii)-pepsin (b) (i)-amylase, (ii)-pepsin, (iii)-trypsin


(c) (i)-trypsin, (ii)-amylase, (iii)-carboxylase (d) (i)-permease, (ii)-carboxylase, (iii)-oxidase

7. Which of the following is most appropriate for aerobic respiration?


Mitochondria Cytoplasm
(a) Glucose   Pyruvate   CO2 + H2O + Energy
Cytoplasm Mitochondria
(b) Glucose   Pyruvate   CO2 + H2O + Energy
Cytoplasm Mitochondria
(c) Glucose   Pyruvate + Energy   CO2 + H2O
Cytoplasm Mitochondria
(d) Glucose   Pyruvate + Energy   CO2 + H2O + Energy
8. A person can choke when a piece of food becomes lodged in the windpipe, blocking the flow of
air. A first aid procedure to remove the blockage is the Heimlich manoeuvre described below:

By performing this procedure, the piece of food is pushed out of the windpipe. Which of the
following causes this to happen?
Prepared by: M. S. KumarSwamy, TGT(Maths) Page - 2 -
(a) The expansion of the chest
(b) The air pressed out of the chest
(c) The food pressed out of the stomach
(d) The upward movement of the wall of the food pipe

9. Choose the incorrect match.


(a) A metal used in joining electric wires - Magnesium
(b) A metal whose oxide is soluble in both acids and bases - Zinc
(c) A metal unreactive towards oxygen and dilute acids - Gold
(d) A metal extracted by using electrolytic reduction - Aluminium

10. Choose the balanced equation (s) from the given options.
I. Ca(OH)2 + CO2 → CaCO3 + H2O
II. Fe + CuSO4 → FeSO4 + Cu
III. KClO4− → KCl + 2O2
IV. Cu + 2HNO3 → Cu(NO3)2 + 2NO2 + H2O
(a) I and II only (b) I, II, and III only (c) I, III and IV only (d) All of these

11. Match the terms of Column (A) with these of Column (B):
Column A Column B
A. Olfactory receptors (i) Tongue
B. Thermoreceptors (ii) Eye
C. Gustatory receptors (iii) Nose
D. Photoreceptors (iv) Skin

(a) A–(iii), B–(iv), C–(i), D–(ii) (b) A–(ii), B–(iii), C–(iv), D–(i)
(c) A–(iv), B–(iii), C–(ii), D–(i) (d) A–(i), B–(ii), C–(iii), D–(iv)

12. At the time of short circuit, the electric current in the circuit
(a) very continuously (b) does not change
(c) reduces substantially (d) increases heavily

13. You are given four bulbs of 25 W, 40 W, 50 W and 60 W. Which bulb has the lowest
resistance?
(a) 25 W (b) 50 W (c) 60 W (d) 40 W

14. Which of the following is a recessive trait in garden pea?


I. Green seed colour II. Terminal flower position III. Violet flower IV. Round seeds
(a) I and II only (b) II and III only (c) I, II and III only (d) I, III and IV only

15. The magnetic field lines due to a straight wire carrying a current are
(a) straight (b) circular (c) parabolic (d) elliptical

16. The most important safety method used for protecting home appliances from short-circuiting or
overloading is
(a) earthing (b) use of fuse (c) use of stabilizers (d) use of electric meter

DIRECTION: In the question number 17 and 20, a statement of Assertion (A) is followed by a
statement of Reason (R).
Choose the correct option
(a) Both assertion (A) and reason (R) are true and reason (R) is the correct explanation of
assertion (A)
(b) Both assertion (A) and reason (R) are true and reason (R) is not the correct explanation of
assertion (A)
(c) Assertion (A) is true but reason (R) is false.
(d) Assertion (A) is false but reason (R) is true.
Prepared by: M. S. KumarSwamy, TGT(Maths) Page - 3 -
17. Assertion (A): Insulin regulates blood sugar level.
Reason (R): Insufficient secretion of insulin will cause diabetes.

18. Assertion: Males have more stature than females during puberty.
Reason: This is because of presence of thyroxin in the blood of females.

19. Assertion (A): If a graph is plotted between the potential difference and the current flowing, the
graph is a straight line passing through the origin.
Reason (R): The current is directly proportional to the potential difference.

20. Assertion (A): Copper sulphate crystals are wet because it contains water of crystallisation.
Reason (R): Water of crystallisation is the fixed number of molecules of water present in one
formula unit of salt.

SECTION – B
Questions 21 to 25 carry 2 marks each.
21. What happens when:
(i) ZnCO3 is heated in the absence of oxygen?
(ii) a mixture of Cu2O and Cu2S is heated?
OR
In the electrolysis of water:
(a) Name the gas collected at the cathode and anode respectively.
(b) Why the volume of one gas, collected at one electrode is double than that of the other
electrode? Name this gas.

22. What are the differences between the transport of materials in xylem and phloem?

23. What are chromosomes? Explain how in sexually reproducing organisms the number of
chromosomes in the progeny is maintained.

24. A student needs spectacles of power – 0.5 D for the correction of his vision.
(a) Name the defect in vision the student is suffering from.
(b) Find the nature and focal length of the corrective lens.
OR
How does the refractive index of earth’s atmosphere vary with height?

25. (i) What is meant by garbage? List two classes into which garbage is classified.
(ii) What do we actually mean when we say that the “enzymes are specific in their action”?
OR
What is an ecosystem? Name the organisms belonging to the second and fourth trophic levels in
the food chain comprising the following:
Frogs, Plants, Snakes, Hawks, Insects

26. Two green plants are kept separately in oxygen free containers, one in the dark and the other in
continuous light. Which one will live longer? Give reasons.

SECTION – C
Questions 27 to 33 carry 3 marks each.
27. What is meant by functional group in carbon compounds? Write in tabular form the structural
formula and the functional group present in the following compounds:
(i) Ethanol
(ii) Ethanoic acid

Prepared by: M. S. KumarSwamy, TGT(Maths) Page - 4 -


28. A compound ‘A’ is used in the manufacture of cement. When dissolved in water, it evolves a
large amount of heat and forms compound ‘B’.
(i) Identify A and B.
(ii) Write chemical equation for the reaction of A with water.
(iii) List two types of reaction in which this reaction may be classified.

29. A squirrel is in a scary situation. Its body has to prepare for either fighting or running away.
State the immediate changes that take place in its body so that the squirrel is able to either fight
or run?
OR
Write in tabular form the location and function of the hormones secreted by each of the
following glands present in the human body:
(i) Pituitary gland
(ii) Thyroid gland
(iii) Pancreas

30. An object is placed at a distance of 10 cm from a convex mirror of focal length 5 cm.
(i) Draw a ray diagram showing the formation of image.
(ii) State two characteristics of the image formed.
(iii) Calculate the distance of the image from mirror.

31. (a) Nichrome wire of length ‘L’ and radius ‘R’ has resistance of 10 Ω. How would the
resistance of the wire change when:
(i) Only length of the wire is doubled?
(ii) Only diameter of the wire is doubled? Justify your answer.
(b) Why element of electrical heating devices are made up of alloys?
OR
State any three differences between Resistance and Resistivity.

32. (i) We do not clean ponds or lakes, but an aquarium needs to be cleaned regularly. Why?
(ii) Why is ozone layer getting depleted at the higher levels of the atmosphere?
Mention one harmful effect caused by its depletion.

33. A 10 cm tall object is placed perpendicular to the principal axis of a convex lens of focal length
12 cm. The distance of the object from the lens is 18 cm. Find the nature, position and size of
the image formed.

SECTION – D
Questions 34 to 36 carry 5 marks each.
34. Define pollination. Explain the different types of pollination. List two agents of pollination.
How does suitable pollination lead to fertilisation?
OR
(a) Draw a diagram of human excretory system and label the following parts on it:
(i) Right renal artery
(ii) Vena cava.
(iii) Urinary bladder
(iv) Left Kidney
(b) List two vital functions of kidney.

35. Calculate the total resistance of the circuit and find the total current in the circuit.

Prepared by: M. S. KumarSwamy, TGT(Maths) Page - 5 -


36. (a) Write chemical equations for the following reactions:
(i) Calcium metal reacts with water.
(ii) Cinnabar is heated in the presence of air.
(iii) Manganese dioxide is heated with aluminium powder.
(b) What are alloys? List two properties of alloys.
OR
Define a chemical reaction. State four observations which help us to determine that a chemical
reaction has taken place. Write one example of each observation with a balanced chemical
equation.
SECTION – E(Case Study Based Questions)
Questions 37 to 39 carry 4 marks each.
37. Case Study - 1
The primary reason behind the formation of the toxic foam is high phosphate content in the
wastewater because of detergents used in dyeing industries, dhobi ghat and households.
Yamuna’s pollution level is so bad that parts of it have been labelled ‘dead’ as there is no
oxygen in it for aquatic life to survive.

Study the passage and answer the questions that follow:


(i) Give an idea about the pH value of the water of river Yamuna if the reason of froth is high
content of detergents dissolved in it. What does it tell us about the concentration of hydroxide
ion and hydronium ion? [2]
(ii) The table provides the pH value of four solutions P, Q, R and S
Solution pH value
P 2
Q 9
R 5
S 11
Arrange the solutions in increasing order of their hydronium ion concentration. [2]
OR
(ii) What is the effect of high content of phosphate ion in river Yamuna? [2]
38. Case Study – 2
Reflex action is a rapid, automatic response to a stimulus which is not under the voluntary
control of the brain. Thus, a reflex action is the one which we perform automatically. It is a
comparatively simple form of behaviour in which the same stimulus produces the same
Prepared by: M. S. KumarSwamy, TGT(Maths) Page - 6 -
response every time. If we unknowingly touch a hot plate, we immediately move our hand away
from it. Moving our hand away on touching a hot plate is an example of reflex action. In a
reflex action, we are unaware of what is going to happen to us and are done without thinking.
How do we respond to it? We respond to it by the process of detecting the signal or the input
and responding to it by an output action. Such a connection is commonly called a reflex arc.
Nerves from all over the body meet in a bundle in the spinal cord on their way to the brain.
Reflex arcs are formed in this spinal cord itself, although the information input also goes on to
reach the brain.

The given figure represents the path of message from the receptor to the effector.
(i) In the given figure, identify “X” and its function. [1]
(ii) Define reflex arc. [1]
(iii) Explain four stages in a reflex action using the above figure. [2]
OR
(iii) Sweating and breathing are not reflex actions. Justify this statement. [2]

39. Case Study – 3


The diagram given below shows an object O and its image I. Without actually drawing the ray
diagram state the following:

(a) Type of lens (Converging / Diverging) [1]


(b) Name two optical instruments where such an image is obtained. [1]
(c) List three characteristic of the image formed if this lens is replaced by a concave mirror of
focal length f and an object is placed at a distance f/2 in front of the mirror. [2]
OR
List four precautions which a student should observe while determining the focal length of a
given convex lens by obtaining image of a distant object on a screen. [2]

Prepared by: M. S. KumarSwamy, TGT(Maths) Page - 7 -

You might also like